Você está na página 1de 134

Universidade Federal de São João Del Rei

PROFMAT 2021

Resoluções das questões do livro de Geometria


Coleção PROFMAT - 2013

ANA CLÁUDIA HOTT PAIVA RICARDO[ACR]


CÉRVULO AUGUSTO[CA]
IVANI MENDES BRITO[IMB]
LUCAS OLIVEIRA DE CARVALHO[LOC]
MARCONI HELBERT VIEIRA [MHV]
NATHÁLIA CÁSSIA LEAL DE DEUS[NLD]
PAULINETTE SILVA MACEDO[PSM]
RONALDO DA COSTA PEREIRA[RCP]
SIDNEI LUÍS DA SILVA[SLS]
VANDERLI DA TRINDADE[VT]

Ouro Branco -MG. December 15, 2021


Alunos da turma anterior

ADAILTON JOSÉ DA SILVA[AJS]


ADONAIA APOLINARIO[AA]
CLARICE AUGUSTA REZENDE DE OLIVEIRA[CAR]
ELIAS VIEIRA DE OLIVEIRA[EVO]
JACIARA CRISTINA DA ROCHA SILVA RODRIGUES[JCR]
JONILSON DOS SANTOS OLIVEIRA[JSO]
JOSÉ EDUARDO BARBOSA DE OLIVEIRA[JEO]
RENATA GONÇALVES DE ALMEIDA[RGA]
TERESA CRISTINA FIRMO GUARANI[TCF]
JOSÉ GERALDO DOS SANTOS BARBOSA[JGB]
WENDEL DE ALMEIDA MAIA[WAM]
EDSON PEREIRA ARRUDA FILHO[EPA]
LUIZ FERNANDO PINHEIRO RAMOS[LFP]
RONALDO DA COSTA PEREIRA[RCP]

2
Soluções dos exercícios do capítulo 1, página 8

Exercício 1.1 [AJS] Sejam A,B, C e D pontos sobre a reta r. Quantas são as semirretas contidas na reta r e
tendo por origem um de tais pontos?

Solução: Como temos 4 pontos, as semirretas são 8, pois de cada ponto partem duas semirretas em
direções opostas.

Exercício 1.2[ACR] Os pontos A,B e C estão todos situados sobre uma mesma reta r, com C ∈ AB . Se
AB = 10cm e AC = 4BC , calculeAC .

Solução: Observe a figura abaixo:

Sendo AC = 4BC , então o segmento AB será dividido em cinco partes iguais a BC. Suponha que, BC = x e
AC = 4x então teremos que : Se AC + BC = AB então : x + 4x = 10 → x = 2cm.

−→ −−→
Exercício 1.3[AA] Sejam A, B, C e D pontos de uma reta r, tais que D ² AC \ AC , B ² DC \ C D e AB = C D.
Prove que AB = C D.

Solução: AC = AB + BC e B D = BC +C D como
AC = B D temos que,
AB + BC = BC +C D anulando BC em ambos os lados temos que
AB = C D.

Exercício 1.4, página 8[CA] Sobre uma reta r estão marcados três pontos A, B e C , tais que B está entre A
e C , AB = 3 cm, AC = 5, 5 cm. Usando somente um compasso, marque sobre r um ponto D entre A e B , tal
que AD = BC .

3
Solução: 1. Centre a ponta seca do compasso em B e fixe a outra extremidade em C .
2. Em seguida, mantendo a mesma abertura aferida no item anterior, centre a ponta seca em A, aponte a
outra extremidade na direção da reta que possui os pontos B e C e marque o ponto D sobre r .
3. Dado que AB > BC , temos que o ponto D estará entre A e B , uma vez que o segmento AD é o segmento
BC transportado, ou seja, AD = BC .

Exercício 1.4

Exercício 1.5[CAR] Marque no plano, com auxílio de uma régua e compasso, três pontos A, B e C tais que
←−
AB = 5 cm, AC = 6 cm e BC = 4 cm.

Solução: Com auxílio de uma régua, faz-se os segmentos AB, AC e BC, de 5 cm, 6 cm e 4 cm respetivamente.
Marca-se no plano o ponto A. Centrando o compasso em A e abertura AB, marca-se o ponto B no plano e
traça-se o segmento AB. Em seguida, centrando o compasso em A e abertura AC, faz-se um arco de centro
A e raio AC. Em seguida, com o compasso centrado em B e abertura BC, faz-se um arco de centro B e raio
BC. A intersecção dos dois arcos é o ponto C.

4
Soluções do cápitulo 1 , página 14

Exercício 1.1, página 14[IMB] Se a interseção de duas regiões convexas de um plano não for um conjunto
vazio, prove que ela também é uma região convexa.

Solução: Se α é convexo, logo, AB ∈ α.


Se β é convexo. logo, AB ∈ β. Suponha que, AB ∈ α ∩ β , de fato o ponto A, B ∈ α então A,B ∈ β. Uma vez
que, α e β são convexos conclui-se que o segmento AB pertende a uma região convexa.

Exercício 1.1

Exercício 1.2, página 14[LOC] Calcule a medida do ângulo que, somado ao triplo de seu complemento,
dá 210° como resultado.

Solução Seja α o ângulo procurado e β seu complementar, temos o seguinte sistema:

α + β =90°
α + 3β =210°

portanto, isso nos resulta em β =60° e α =30°

Exercício 1.3, página 14[EVO] Calcule as medidas de dois ângulos complementares, sabendo que o complemento
do dobro de um deles é igual à terça parte do outro.

Solução Sejam α e β dois ângulos tais que β = 90◦ − α e o complemento do dobro de α é igual à terça
parte de β, ou seja,
90◦ − α
90◦ − 2 α = .
3
α
Portanto, 2 α − = 90◦ − 30◦ e os ângulos são α = 36◦ e β = 54◦ .
3

γ β
Solução: Pela informação dada no enunciado da questão sabemos que : α = 6
+ 2
( I ). Como α e β são
opostos pelo vértice segue que: α = β (II). Sabemos também que α e β são suplementares, logo: α+β = 180◦
(III) e γ = 180◦ − α (IV).
Substituindo (II) e (IV) em (I) temos que: α = 180−α
6
+ α2 → α = 180
4
→ α = 45◦

5
Exercício 1.4, página 14[MHV] Os ângulos α e β são OPV e suas medidas em graus são expressas por
9x − 2 e 4x + 8, respectivamente . Calcule, também em graus a medida de α + β.

Exercício 1.4

Solução: Sabendo que α e β são OPV e suas medidas são 9x-2 e 4x+8 , respectivamente, tem -se: α = β,
pois são OPV,então: 9x − 2 = 4x + 8 → 9x − 4x = 8 + 2 → 5x = 10 → x = 2. Logo α = β = 15◦ e α + β = 32◦ .

Exercício 1.5 , página 14[JCR] Se duas retas se intersectam, prove que um dos ângulos por elas formados
é igual a 90◦ se, e só se, os quatro ângulos o forem.

Solução

←→ ←→
Sejam AB e C D duas retas concorrentes no ponto O e o AÔC = 90◦ , temos que:
AÔC +C ÔB = 180◦ , pois formam um ângulo raso. Logo C ÔB = 90◦ .
Os ângulos AÔC e B ÔD são ângulos opostos pelo vértice, portanto são congruente. Assim B ÔD = 90◦ .
E, AÔC e AÔD também formam um ângulo raso. Logo AÔD = 90◦
Portanto os quatro ângulos formados pelas retas são de 90◦ .
Por outro lado se AÔC 6= 90◦ , como AÔC + C ÔB = 180◦ , pois formam um ângulo raso, temos que: C ÔB 6=
90◦ .
Os ângulos AÔC e B ÔD; e AÔD e B ÔC são ângulos opostos pelo vértice, portanto são congruente. Assim
B ÔD 6= 90◦ e AÔD 6= 90◦ .

Exercício 1.6[NCD] Na figura abaixo, o ângulo α mede a sexta parte do ângulo γ mais a metade do ângulo
β. Calcule a medida de α.

6
γ β
Solução: Pelo enunciado temos que : α = 6 + 2 . Como α e β são opostos pelo vértice segue que, α = β →
γ γ
α = 6 + α2 → α = 3 . Sabemos que α + γ = 180◦ . Logo, α = 180−α
3
→ α = 45◦ .

Exercício 1.7, página 15[JSO] Cinco semirretas, de mesma origem O, formam cinco ângulos que cobrem
todo o plano e têm medidas em graus proporcionais aos números 2, 3, 4, 5 e 5. Calcule a medida do mais
de tais ângulos.

ai
Solução: Sejam as frações , , ..., bann
a1 a2
b1 b2
todas proporcionais, então bi
= a 1 +a 2 +...+a n
b 1 +b 2 +...+b n
. dada a figura abaixo

Seguindo o teorema acima temos:

2+3+4+5+6
b 2 +b 3 +b 4 +b 5 +b 6
= b22
20 2
360 = b 2
b 2 = 36
Seguindo o mesmo raciocínio temos b 3 = 54, b 4 = 72, b 5 = 90 e b 6 = 108

Exercício 1.8[PSM] Construa , com régua e compasso, um ângulo cuja medida seja igual a soma das
medidas dos ângulos ∠ AOB e ∠ A 0O 0 B 0 da figura abaixo:

7
−−→
Solução: Primeiramente traçamos a semi-reta O A, passando por A. Agora façamos uma circunferência
com a ponta fixa em O; para podermos copiar ( transcrever o ângulo AÔB .

Exercício 1.8

Exercício 1.9, página 15[JEO] Sobre um círculo de centro O marcamos três pontos A, B e C , tais que AB
= BC = AC , onde os arcos a que nos referimos são os arcos menores correspondentes. Calcule as medidas
dos ângulos ∠ AOB , ∠BOC , ∠ AOC .

Solução:

O circulo Γ possui ângulo todal de 360◦ . Traçando os segmentos O A, OB e OC , formamos os arcos AB ,


BC e AC cujos ângulos correspondentes são α, β e γ respectivamente e todos eles fração do ângulo total da
circunferência. Como AB = BC = AC , a fração correspondente do ângulo total da cirfunferência será igual
para todos eles, logo, α = β = γ = 120◦ .

Exercício 1.10, página 15[RCP] 1Calcule a medida de uma ângulo agudo, sabendo que a mesma excede
a medida de seu complemento em 76◦ .

8
Solução: Sabendo que α é agudo, tem-se:
β + α = 90ř −→ β = 90ř − α de onde:
α − (90ř − α) = 76◦ −→ α − 90ř + α = 76◦ −→ 2α = 166ř
Logo, α = 83ř.

Exercício 1.11, página 15[JGB] Três semirretas de mesma origem O formam três ângulos que cobrem
todo o plano. Mostre que ao menos um desses ângulos mede pelo menos 120◦ e ao menos um mede no
máximo 120◦ .

Solução:

Exercício 1.11
−−→ −−→ −−→
Dadas as semirretas O A, OB e OC de mesma origem O e sabendo que as mesmas cobrem todo o plano,
podemos afirmar que α + β + θ = 360◦ . Suposição 1: Nenhum ângulo é maior, ou igual 120◦ . Logo é um
absurdo, pois a soma dos três ângulos seria menor que 360◦ . Suposição 2: Nenhum ângulo é menor, ou
igual a 120◦ . Também é um absurdo, a soma desses três ângulos ultrapassaria 360◦ . Portanto, ao menos
um desses ângulos deverá medir pelo menos 120◦ e ao menos um desses ângulos deverá medir no máximo
120◦ .

9
Soluções do capítulo 1, página 20

Exercício 1.1, página 20[JGB] Prove a fórmula para o número de diagonais de um polígono convexo
(preposição) por indução sobre om número de lados do mesmo.

n(n−3)
Solução: Queremos provar que, todo n-ágono convexo possui exatamente 2 diagonais, n ≥ 4.

Usando a indução finita, temos:

4.(4−3)
i) Para n = 4, verifica-se pois todo quadrilátero possui 2 = 2 diagonais.

k.(k−3)
ii) Supondo que vale para um polígono A1 A2 ... Ak , com k lados, ou seja, dk = 2
.(*)

(k+1).(k−2)
Vamos mostrar que vale para um polígono A1 A2 ... Ak Ak +1 lados, isto é: dk +1 = 2 .

Pela hipótese de indução (*), ao acrescentarmos mais um lado no polígono de k lados, ele terá k + 1
lados.
E assim, as diagonais do polígono são: A1 A3 , A1 A4 ,..., A1 Ak +1 .
Podemos dividir em três tipos de diagonais:

a) A1 Ak ;
b) diagonais de A1 A2 ... Ak ;
c) diagonais de Ai Ak +1 , para 2 ≤ i ≤ k - 1.

Somando esses diferentes tipos de diagonais, obtemos:


(k).(k−3) 2+k(k−3)+2(k−2)
dk +1 = 1 + 2 + (k - 2) = 2 + (k - 2) =

k 2 −3k+2k−2 2 2
= 2
= k −k−2
2
= k −2k+k−2
2
=
k.(k−2)+(k−2) (k+1).(k−2)
= 2
= 2
.

Exercício 1.2, página 20:[SLS] A partir de um dos vértices de um polígono convexo podemos traçar tantas
diagonais quantas são as diagonias de um hexágono. Encontre o número de lados do polígono.

Solução: O número de diagonais de um hexágono ( 6 lados) é dado por : d = 6.(6−3)


2 = 9.
Assim, se cada vértice desse polígono, partem exatamente 9 diagonais.
Por outro lado, de cada vértice de um polígono com n-lados partem n-3 diagonais, pois desconta-se sempre
os dois lados consecutivos , da direita e da esquerda e como o vértice dado não pode ser unido a ele mesmo,
temos n-3. Daí, n − 3 = 9 ⇒ n = 12. Logo , o polígono dado é um dodecágono.

10
Exercício 1.3, página 20[RGA] Três polígonos convexos têm números de lados iguais a três naturais consecutivos.
Sabendo que a soma dos números de diagonais dos polígonos é 133, Calcule o número de lados do polígono
com maior número de diagonais.

Solução: Considere o número de lados de cada polígono representados respectivamente por x, x + 1 e x +


2. Como a soma de todas as diagonais é igual a 133, temos:

x.(x − 3) (x + 1)(x + 2) (x + 2)(x − 1)


+ + = 133
2 2 2

x 2 − 3x + x 2 − x − 2 + x 2 + x − 2 = 266

3x 2 − 3x − 270 = 0

x 2 − x − 90 = 0

∆ = (−1)2 − 4.1.(−90) = 361


1 + 19 20
x= = = 10
2 2

Logo os polígonos são o decágono (10 lados), undecágono (11 lados) e dodecágono (12 lados).
Portanto o polígono com maior número de diagonais é o dodecágono.

11
Soluções dos exercícios do capítulo 2

Exrcício 2.1[TCF] Construa com régua e compasso as bissetrizes internas do triângulo ABC da figura 2.7.

Solução: Coloque a ponta do compasso no vértice A e trace um arco que intercepte ambos os lados do
ângulo. Coloque a ponta do compasso sobre um dos pontos em que o primeiro arco intercepta um dos
lados do ângulo e desenhe um segundo arco em algum lugar na região contida entre os lados do ângulo.
Coloque a ponta do compasso no ponto de interseção do primerio arco com o outro lado do ângulo sem
alterar a abertura do compasso. Desenhe um terceiro arco que intercepte o segundo arco em algum ponto.
Desenhe uma reta que ligue o vértice do ângulo e o ponto de interseção do segundo e terceiro arcos. Repita
todo o processo para os vértices B e C .

Exercício 2.1

Exercício 2.2, página 34:[VT] Construa com régua e compasso as medianas do triângulo ABC da figura
2.8.

Solução: Para construção do triângulo ABC, onde são conhecidos AB = c, AC = b e m a da sua mediana
relativa a BC, seguiremos os passos:
1) Tracemos uma reta r e nela marquemos os pontos A e B, tais que AB = c;
2) Com o compasso, tracemos dois círculos: Γ1 com entro em B e raio igual a c e Γ2 com centro em A e raio
igual a b;
3) Na interseção de Γ1 e Γ2 marquemos o ponto C;
4)Com uma régua, tracemos AC = b e BC = a;
5) Com o compasso, tracemos o círculo Γ3 com centro em C e raio igual a a;
6) Pelas interseções de Γ3 e Γ1 tracemos a reta s;
7) Na interseção de s com BC, marquemos o ponto M , que é ponto médio de BC;
8) Tracemos, de A a M, a mediana AM = m a .

12
Exercício 2.4[NLD] Sejam dados, no plano, um ponto A e uma reta r , com A 6∈ r . Dizemos que um ponto
A 0 é o simétrico de A em relação á reta r quando A A 0 ⊥ r e r passar pelo ponto médio do segmentoA A 0 .
Mostre como construir A 0 com régua e compasso.

Solução: Construção do simétrico do Ponto A Dado: uma reta r e um ponto A não pertencente a reta r.
Passos:
• Tome uma reta r e um ponto A fora dela.
•Destaque um ponto B ∈ r e uma abertura B A , trace a cincunferência de raio B A e centro B , denominada
C1 .
•Trace uma cincunferência de raio C A e centro c denominada C 2.
• O ponto de interseção de C 1 com C 2 é simétrico de A.

Construção do Exercício 2.4

AB ∼
Justificativa: d =Ad ˆ ∼
0C possuem o mesmo ângulo central. A BC ˆ e os triângulos 4ABC ∼
= A 0 BC = 4A 0 BC .
O 4A 0 B A é isósceles e BC é bissetriz de A A 0 .

Exercício 2.5[AJS] Construa com régua e compasso o triângulo ABC , conhecidos os comprimentos AB =
c, e BC = a e m a da mediana relativa a A. (Sugestão: comece construindo o triângulo ABC , onde M é o
ponto médio do lado BC.)
Solução. Descrição dos passos: Tomemos os comprimentos a seguir arbitrados.

13
1. Trace uma reta r , sobre a mesma, marque os pontos B e C tais que BC = a.

2. Fixe uma abertura d > 21 BC e trace, dois círculos de raio r e centros B e C , arcos que se intersectem
nos pontos nos pontos X e Y .
←→
3. Marque o ponto médio M interseção da reta X Y com o segmento BC .

4. Fixe uma abertura em B e construa um círculo α de raio AB = c.

5. Fixe uma abertura em M e construa um círculo β de raio AM = m a .

6. Marque o ponto A interseção dos círculos α e β.

7. Trace os segmentos AB , AM e AC .

Exercício 2.5

Exercício 2.7[AA] Construa com régua e compasso o triângulo ABC , conhecidos os comprimentos AB =
c e βa da bissetriz interna relativa ao lado BC , bem como a medida ∠B AC = α.

Solução:

1. Trace o segmento AB de comprimento c;

2. Em A, construa o ângulo ∠B AB 0 = α;
α
3. Trace a bissetriz s do ângulo ∠B AB 0 ; assim B AS b 0=
b = S AB
2

14
4 Marque em s o ponto D, tal que AD = B a ;
−−→ −→ −−→
5 Trace ² B D, obtendo C = AB ∩ B D

Exercício 2.8, página 35[CA] Se ABC é um triângulo isósceles de base BC , prove que a bissetriz, a mediana
e a altura relativas a BC coincidem.

Solução:
Seja M o ponto médio do lado BC do triângulo ABC . Em relação aos triângulos AB M e AC M , dado
que ABC é isósceles de base BC , temos que AB = AC e Bb = Cb. Como M é ponto médio, verifica-se que
B M = C M , logo, segue do caso LAL que AB M ≡ AC M .

Do fato de AM ser comum aos triângulos e oposto aos ângulos Bb e Cb, segue que os ângulos opostos a
B M e C M , respectivamente, são congruentes. Portanto, B
ƒ AM = C
ƒ AM . Sendo assim, como a mediana AM
divide Ab em dois ângulos iguais, tem-se que a mediana coincide com a bissetriz do referido ângulo.

Análogo ao apresentado acima, os ângulos opostos aos lados AB e AC , respectivamente, também são
congruentes, ou seja, AM
ƒ B = AMC
ƒ . Como AMƒ ƒ = 180◦ e os ângulos têm igual medida, conclui-se
B + AMC
que AM
ƒ ƒ = 90◦ . Logo, M é o pé da perpendicular ←
B = AMC
−→
AM sobre a base BC . Portanto, AM é também a
altura relativa ao lado BC .

15
Assim, em um triângulo isósceles ABC de base BC , tem-se que bissetriz, mediana e altura relativas a BC
coincidem.

Exercício 2.9:[CAR] Sejam ABC um triângulo e P , M e H respectivamente os pés da bissetriz interna,


mediana e altura relativas ao lado BC . Se P e H ou M e H coniciderem, prove que ABC é isóceles de base
BC .

Solução: Sejam P e H coincidentes: como P é o pé da bissetriz interna relativa ao lado BC, AP é bissetriz
de ∠B AC , logo, ∠B AP = ∠P AC . Como H é a altura relativa a BC, temos que ∠B HC = ∠ AHC = 90◦ .
Analisando os triângulos AHB e AHC temos que α = β, o lado AH comum, e ∠B HC = ∠ AHC . Portanto,
os triângulos são semelhantes pelo caso ALA e, consequentemente, AB = BC, fazendo com que o triângulo
ABC seja isóceles de base BC.

Sejam M e H coincidentes: como M é o pé da mediana relativa a BC, têm-se que BM = MC. Como H
é o pé da altura , então, ∠B HC = ∠ AHC = 90◦ . Se M = H, então, analisando os triângulos AMB e AMC,
temos que ∠B HC = ∠ AHC , BM = MC e AM é uma lado comum. Portanto, os triângulos são semelhantes
pelo caso LAL e, consequentemente, AB = BC, fazendo com o que o triângulo ABC seja isócelses de base BC.

16
Exercício 2.10[IMB] Seja Γ um círculo de centro O e AB uma corda de Γ. Se M é um ponto sobre AB,
prove que:

OM ⊥AB → AM = B M

Solução: Se OM ⊥AB então temos a construção de dois triângulos a partir dos raios OA e OB. No triângulo
BMO reto em M temos os lados comuns O A = OB a hipotenusa dos triângulos.
Pelo caso especial de congruência de triângulos retângulos podemos concluir que o triângulo AMO e BMO
são congruentes. De fato AM = BM sendo AM=BM e OA=OB pois OA e OB são os raios da cincinferência r.
Como OM é um lado comum para os dois triângulos AMO e BMO e pelo caso LAL os triângulos são
congruentes. Sendo assim M o ponto médio da corda AB unida com o seguimento MO formam um ângulo
de 90◦ , logo AM = B M → OM ⊥AB .

17
Soluções dos exercícios do cápitulo 2, página 42

Exercício 2.1, página 42 [LOC] Se dois triângulos retângulos são tais que a hipotenusa e um dos catetos
do primeiro são respectivamente congruentes à hipotenusa e a um dos catetos do outro, prove que os
triângulos são congruentes.

Solução Considere os triângulos ABC e A 0 B 0C 0 .

Sabemos que BC = B 0C 0 sem perda de generalidade consideremos que AC = A 0C 0

Transportemos o ∆A 0 B 0C 0 de modo que a hipotenusa B 0C 0 coincida com a hipotenusa BC , ficando o vértice


←→
A 0 no semiplano oposto ao vértice A, em relação a reta BC .

Unimos os pontos A e A 0 cujo segmento interceptará a reta suporte de lado BC .

Dessa construção e sendo AC = A 0C 0 , temos que o ∆A 0C A é isosceles e portanto, os ângulos C Ab A 0 =


A0 A
Cc

A 0 = 90° e A 0C 0 = AC .
Como Ab = c

Podemos concluir que A 0 D = AD e portanto, os ∆A 0 DC 0 ≡ ∆ADC , logo:

A 0C
c0 D = ACbD ⇒ Cb = C
c0 .

Portanto, os triângulos ABC e A 0 B 0C 0 são congruentes pelo critério ALA.

Exercício 2.2 página 43 [MHV] ABC é um triângulo isósceles de base BC e D ∈ AB, E ∈ AC são pontos tais
←→ ← →
que DE ∥ BC . Sendo F o ponto de interseção dos segmentos CD e BE , mostre que B F = C F .

18
Solução:

Exercício 2.2
←→ ← →
ABC é isósceles DE ∥ BC queremos mostrar que B F = C F . Como DE ∥ BC , temos A Ê D = AĈ B , pois são
correspondentes e A D̂E = A B̂C , pois também são correspondentes. Do fato de ABC ser isósceles , então
A B̂C = AĈ B ,daí A Ê D = A D̂E , ou seja, o triângulo ADE é isósceles , de base DE.
Assim, como AB = AC , segue que B D = C E . Daí, B D = C E , D B̂C = E B̂C e BC é lado comum aos triâgulos
DBC e ECB, logo pelo caso LAL, concluímos que os triângulos DBC e ECB são congruentes .
Logo, em particular os ângulos D Ĉ B e E B̂C são congruentes , isto acarreta BFC é isósceles de base BC.
Portanto , B F = C F .

Exercício 2.3.Página 42[EVO] Seja ABC um triângulo isósceles de base BC . Prove que as alturas, medianas
e bissetrizes internas relativas aos lados AB e AC têm comprimentos iguais.
Solução:

i)

Os pontos D e E são, respectivamente, os pés das alturas relativas aos lados AC e AB. Queremos mostrar
que as alturas relativas aos lados AC e AB do triângulo isósceles ABC, de base BC, são iguais.
Os triângulos ABE e ACD são congruentes pelo caso LAAo , pois

19
• AB = AC

• ∠ ADC = ∠ AE B

• ∠C AD = ∠B AE .

Então, em particular, C D = B E , ficando demonstrado que as alturas relativas aos lados AC e AB do triângulo
isósceles de base BC são iguais.

ii)

Os pontos M e N são, respectivamente, os pontos médios dos lados AB e AC. Queremos mostrar que as
medianas MC e N B relativas aos lados AC e AB do triângulo isósceles ABC são iguais.
Os triângulos MBC e NCB são congruentes pelo caso LAL, pois

• M B = NC

• ∠M BC = ∠NC B

• BC = BC .

Então, em particular, MC = N B , ficando demonstrado que as medianas relativas aos lados AC e AB do


triângulo isósceles de base BC são iguais.

iii)

20
Os pontos P e Q são, respectivamente, os pés das bissetrizes internas relativas aos lados AC e AB.
Queremos mostrar que as bissetrizes internas relativas aos lados AC e AB do triângulo isósceles ABC, de
base BC, são iguais.
Os triângulos BCP e CBQ são congruentes pelo caso ALA, pois

• ∠PC B = ∠QBC

• BC = BC

• ∠P BC = ∠QC B.

Então, em particular, PC = QB , ficando demonstrado que as bissetrizes internas relativas aos lados AC e
AB do triângulo isósceles de base BC são iguais.

Exercício 2.6[PSM] Na figura abaixo, prove que r ∥ s ⇐⇒ α = β ( os ângulos α e β são deniminados


correspondentes.

Exercício 2.6

21
Solução: Observamos que β0 é oposto pelo vértice de β, portanto β = β0 inicialmente, note que, como
θ + β0 = 180◦ , temos α = β ⇐⇒ α + θ = 180◦ . Portanto, basta provarmos que r ∥ s ⇐⇒ α = β0 . Verificamos
que se α = β0 ⇒ r ∥ s, de modo que basta provar a implicação contrária. Suponha que, r ∥ s. Então s é a
única reta paralela a r e passando por B, de sorte que pode ser construída, logo α = β0 .

ˆ = 20ř e DE
Exercício 2.8, página 44[RCP] 2.8 Na figura abaixo, temos ABC ˆ F = 25ř. Sabendo que as retas
←→ ← → ˆ .
AB e E F são paralelas, calcule a medida do ângulo C DE

Solução:

←→ ← →
Dadas as retas r e s paralelas a AB e E F temos:
←→
. De AB //s, b = 20ř
←→
. De E F //r , c = 25ř
. De r //s, a = d
Mas , a + b = 60ř −→ a + 20ř = 60ž −→ a = 40ř = d .
Como c + d = x −→ x = 25ř + 40ř −→ x = 65ř.
ˆ = 65ř
Logo, C DE

22
Exercício 2.10, página 44:[SLS] Calcule a soma dos ângulos nos vértices A,B,C,D e E da estrela de cinco
pontas da figura abaixo:

Ab + Bb + Cb + Eb = θ. Sejam X o ponto de interseção de AB e C D e Y o ponto de interseção de AE e C D.

Pelo Teorema do ângulo externo em BCX , temos:


x = Bb + Cb.
•b
• Em DEY , temos yb = D b + Eb. Por outro lado, xb + yb + Ab = 180◦ ⇒ x + y = 180◦ − A.
b Daí , xb + yb = Bb + Cb + D
b + Eb ⇒
180◦ − Ab = Bb + Cb + D b + Eb = 180◦ . Logo, θ = 180◦ .
b + Eb ⇒ Ab + Bb + Cb + D

Exercício 2.12, página 44:[VT] Em um triângulo ABC, seja M o ponto médio do lado BC. Se AM = 12 BC ,
b = 90◦ .
mostre que B AC

23
Solução: M é o ponto médio de BC , AM = 12 BC . Queremos mostrar que B AC
b = 90◦ . Se AM = 1 BC , então
2
AM = B M = C M , assim os triângulos ABM e ACM são isósceles, com bases respectivamente AB e AC . Daí,
b = α e C AM
A BbM = B AM b = ACbM = β. Fazendo a soma dos ângulos: Ab + Bb + Cb = 180◦ → (α + β) + α + β =
180◦ → 2(α + β) = 180◦ → α + β = 90◦ . Logo , Ab = 90◦ .

Exercício 2.16 , página 45[IMB] Um triângulo ABC é isósceles de base BC. Os pontos D e F sobre BC e E
sobre AC são tais que AD = AE e B ÂD = 48◦ . Calcule D̂E .

Solução: Dado um triângulo ABC, isósceles de base BC, tem-se:


• θ = α + β ( pela propriedade do ângulo externo).
• θ + α = β + 48◦ (pela propriedade do ângulo externo).
Logo, substituindo, tem-se: (α + β) + α = β + 48◦ → 2α + 48 → α = 24◦ ou C D̂E = 24◦ .

Exercício 2.17, página 45[IMB] O triângulo ABC é isósceles de base BC. Os pontos D e F sobre o lado AB
e E sobre o lado AC são tais que BC = C D = DE = E F = F A. Calcule a medida do ângulo ∠B AC .

Solução: Dado o triângulo ABC isósceles de base BC e BC = DC = DE = E F = F A.


(1) O triâgulo AEF é isósceles da base E A logo F Ê A = F ÂE = α (Congruentes). Logo o ângulo D F̂ E , externo
ao triângulo AEF é igual a somas destes ângulos: D F̂ E = 2α.
(2 Como o triângulo FED é isósceles de base FD, temos : D F̂ E = F D̂E = 2α.
(3) Sabendo que ÊC é externo ao triângulo AED, temos D ÊC = 3α.
(4) Sabendo que o triângulo ECD é isósceles de base EC, temos : D ÊC = D Ĉ E = 3α.
(5 Tendo que o ângulo B D̂C é externo ao triângulo CDA, temos que : B D̂C = 4α.
(6) Por fim, o triângulo DBC é isósceles de base BD de modo que B D̂C = D B̂C = 4α. Como o triângulo ABC
é isósceles de base BC , os ângulos A B̂C E AĈ B são congruentes, portanto: A B̂C = AĈ B = 4α de onde :
α + 4α + 4α = 180◦ → 9α = 180◦ → α = 20◦ .

Exercício 2.18, página 45[PSM] ABCDEF é um hexágono tal que as diagonais AD, BE e CF passam todas
por um mesmo ponto M, que as divide ao meio. Prove que  + B̂ + Ĉ = 360◦ .

24
Solução: Verifique a figura abaixo:

Observamos que se M divide AD,BE e CF ao meio temos então formados os triângulos 4AB M ∼
= 4DE M
∼ ∼
e 4AF M = 4C D M e o 4BC M = 4E F M todos pelo caso LAL.
Temos então que os ângulos : 2c + 2g + 2 j = 360◦ = 2(c + g + j ) = 360◦ → c + g + j = 180◦ .
Se  = a + e ; B̂ = f + h; Ĉ = i + b.
Alem disso temos :
2(a + b + c) = 360◦ ⇒ a + b + c = 180◦
2(e + f + g ) = 360◦ ⇒ e + f + g = 180◦
2(h + i + j ) = 360◦ ⇒ h + i + j = 180◦ .
Podemos concluir que : a +b +c +d +e + f +g +h +i + j = 540◦ → Â + B̂ + Ĉ = 540◦ −180◦ → Â + B̂ + Ĉ = 360◦ .

25
Soluções dos exercícios do cápitulo 2, página 50

Exercício 2.1, página 50[AJS] Se dois lados de um triângulo isósceles medem 38 cm e 14 cm, calcule seu
perímetro. (Sugestão: use a desigualdade triangular para mostrar que o terceiro lado não pode medir 14
cm.)

Solução: Pela desigualdade triangular, devemos ter 38 < 14 + x, x ∈ {14, 38}, pois o triângulo é isósceles.
Assim, x = 38 e o perímetro é 2p = 2 · 38 + 14 = 90.

Exercício 2.2, página 50[PSM] Encontre o intervalo de variação de x no conjunto dos reais, sabendo que
os lados de um triângulo são expressos em centímetros por x + 10 , 2x + 4 e 20 − 2x.

Solução: Parte (1)


x + 10 < 2x + 4 + 20 − 2x
x + 10 < 24
x < 24 − 10
x < 14.
Parte (2)
2x + 4 < x + 10 + 10 − 2x
2x + 4 < −x + 30
2x + x < 30 − 4
3x < 26
x < 26
3 .
Parte (3)
20 − 2x < x + 10 + 2x + 4
20 − 2x < 3x + 14
−2x − 3x < 14 − 20
−5x < −6(−1)
5x > 6
x > 65 .
Resposta : {x ∈ R| 56 < x < 26
3 }

Exercício 2.3,página 51[AA] Em um triângulo ABC , o lado AB tem por comprimento um número inteiro
de centímetros. Calcule o maior valor possível para AB , sabendo que AC = 27cm, BC = 16cm e que
Ĉ < Â < B̂ .

Solução: Pela desigualdade triangular que a < b + c, logo temos que


27 < 16 + AB =⇒ AB > 27 − 16 =⇒ AB > 11
16 < 27 + AB =⇒ AB > 27 − 16 =⇒ AB > 11

26
AB < 27 + 16 =⇒ AB > 43

Pela hipótese de que Ĉ < Â e BC = 16cm,que corresponde ao lado oposto de A, isso acarreta que
AB < AC , ou seja, AB = 16cm, caso contrário, seria  < Ĉ .
Como AB é inteiro, o seu maior valor será 15cm.

Exercício 2.4, página 51[PSM] Em um triângulo ABC, escolhemos os pontos P ∈ BC , Q ∈ AC e R ∈ AB ,


todos diferentes dos vértices de ABC. Prove que o perímetro do triângulo PQR é menor que o perímetro do
triângulo ABC.

Exercício 2.4

Solução: Observamos que o perímetro do 4ABC é dado por : 2p = AR + RB + B P + PC + CQ + AQ. No


4PQR o perímetro é dado por : 4PQR → 2p = QR + PQ + P R. Verificamos que:
No 4ARQ : RQ < AR + AQ
No 4B RP → RP < RB + B P ,
No 4QPC → QP < QC +PC . Substituindo: RQ +RP +QP < AR + AQ +RB +B P +QC +PC → perímetro do
4RQP < 4ABC .

Exercício 2.5, página 51[CAR] Se a, b, c são os comprimentos dos lados de um triângulo, prove que |b −
c| < a.

Solução: Seja um triângulo de lados a, b, c. Pela desigualdade triangular temos:

a < b +c (1)

b < a +c (2)

c < a +b (3)

27
Mostrar que |b − c| < a
|b − c| < a ⇒ b − c < a ou − (b − c) < a
Rearranjando a expressão 2, temos:
b < a +c ⇒ b −c < a
Rearranjando a expressão 3, temos:
c < b + c ⇒ c − b < a ⇒ −(b − c) < a
Portanto, |(b − c)| < a é válido se a, b, c são comprimentos dos lados de um triângulo.

Exercício 2.6[PCM] Se a,b,c são os comprimentos dos lados de um triângulo , prove que a 3 + b 3 + 3abc >
c 3.

Solução: Pela desigualdade triangular temos: a + b > c e que (a + b)3 > c 3 então temos: (a + b)3 = a 3 +
3a 2 b + 3ab 2 + b 3 então:
a 3 + 3a 2 b + 3ab 2 + b 3 > c 3
a 3 + 3ab(a + b) + b 3 > c 3
a 3 + 3abc + b 3 > c 3 .

Exercício 2.7, página 51[IMB] Dado um quadrilátero convexo ABCD, prove que o ponto P do plano para
o qual a soma P A + P B + PC + P D é mínima é o ponto de concurso das diagonais de ABCD.

Solução:

Exercício 2.7

Dado o quadrilátero ABCD desejamos provar que : P A + P B + PC + P D é minima se P pertence a uma


das diagonais.
(1) Observamos o triângulo APE pela desigualdade triangular temos P A + PC > AC . Para que P A + PC seja
mínima devem ter P A + PC = AC , neste caso P dever pertencer a AC.
(2) Já no triângulo BPD, pela desigualdade triangular , temos:
B P + P D > B D. Logo para que P B + P D = B D, o que só ocorre quando P pertence a B P . Portanto se P deve
pertencer a AC e a BC para que a soma P A + P B + PC + P D seja mínima , então P é o ponto de concurso
das diagonais.

28
Exercício 2.9, página 51[EVO] Na figura abaixo, as semirretas r e s são perpendiculares. Construa com
régua e compasso os pontos B ∈ r e C ∈ s para os quais a soma AB + BC +C D seja a menor possível.

Exercício 2.9

Solução: DESCRIÇÃO DOS PASSOS

1 Com o compasso centrado em A, descreva um arco de círculo que intersecte a semirreta r em dois pontos
distintos E e F.

2 Com o compasso centrado em E, descreva um semicírculo que passe por A.

3 Com o compasso centrado em F, descreva um semicírculo que passe por A.

4 Marque o ponto A’, interseção dos semicírculos descritos nos passos 2 e 3.

5 Com o compasso centrado em D, descreva um arco de círculo que intersecte a semirreta s em dois pontos
distintos G e H.

6 Com o compasso centrado em G, descreva um semicírculo que passe por D.

7 Com o compasso centrado em H, descreva um semicírculo que passe por D.

8 Marque o ponto D’, interseção dos semicírculos descritos nos passos 6 e 7.

9 Trace o segmento de reta A 0 D 0 .

10 Marque os pontos B e C, interseção de A 0 D 0 com as semirretas r e s, respectivamente.

11 Marque o ponto B’, distinto de B, em r.

12 Marque o ponto C’, distinto de C, em s.

29
Queremos mostrar que a soma AB + BC +C D é mínima, ou seja, que

AB + BC +C D < AB 0 + B 0C 0 +C 0 D.

Segue da desigualdade triangular e do fato de A’ e D’ serem simétricos de A e D, respectivamente, que:

• AB + BC +C D =
= A 0 B + BC +C D 0 < A 0 B 0 + B 0 D 0

• B 0 D 0 < B 0C 0 +C 0 D 0

Somando as desigualdades, temos

AB + BC +C D < AB 0 + B 0C 0 +C 0 D,

como queríamos mostrar.

Exercício 2.11, página 51[JCR] Seja ABC um triângulo equilátero de lado l . Se P e Q são pontos situados
respectivamente sobre AB e AC , distintos dos vértices de ABC , prove que BQ + PQ +C P > 2l .

Solução Sejam B 0 o simétrico de B em relação a AC e C 0 o simétrico de C em relação a AB 0 . Se P 0 é o


simétrico de P em relação a AC , temos que PQ = P 0Q e C P = C P 0 = C 0 P 0
Portanto
BQ + PQ +C P = BQ + P 0Q +C 0 P > BC 0 = 2l

30
Soluções dos exercícios do cápitulo 2 , página 68

Exercício 2.1, página 68[JSO] Se dois segmentos são iguais e paralelos, prove que suas extremidades são
os vértices de um paralelogramo.

Solução:

Exercício 2.1

Por definição um quadrilátero convexo é paralelogramo, se e somente se, suas diagonais se interceptam
nos respectivos pontos médios.
Temos AB //C D e AB ≡ C D, então podemos traçar os segmmentos de reta AC eB D onde AC ∩B D = {M }

Os triângulos 4AM B e 4C M D são congruentes pelo caso ALA, como D M = B M e AM = C M temos M


o ponto médio. Logo ABC D é um paralelogramo.

Exercício 2.2, página 68[ACR] Seja ABCD um quadrilátero qualquer. Mostre que os pontos médios de
seus lados são os vértices de um paralelogramo.

Solução: Considere um quadrilátero (reverso, que é o caso mais geral) de vértices A,B,C e D, sendo A,C
e B,D opostos. Trace a diagonal BD, de modo a obter dois triângulos : ABD e BCD, como mostra a figura
abaixo:

Exercício 2.2

Seja M e N os pontos médios dos lados AB e AD, respectivamente. Trace o segmento MN. Note que ele é
base média do triângulo ABD e, então, deve ser paralelo a BD: M N ∥ B D
Trace, agora, o segmento P e Q , sendo P e Q os respectivos pontos médios de BC e CD. Observe que PQ é a

31
base média no triângulo BCD, o que implica que é paralelo a base: PQ ∥ B D.
Pela transitividade do paralelismo de retas, se duas retas são paralelas a uma terceira, então elas são paralelas
entre si. Portando: M N ∥ B D ; PQ ∥ B D → M N ∥ PQ .
Note que MNPQ é o quadrilátero com vértices nos pontos médios nos lados do quadrilátero original. Você
já provou que dois dos lados opostos são paralelos. Usando a mesma analogia para os segmentos PM e NQ
em relação a diagonal AC verificamos que: P M ∥ AC ; NQ ∥ AC → P M ∥ NQ. Assim, o quedrilátero MNPQ
é um paralelogramo.

Exercício 2.3 , página 68[JEO] Uma reta r passa pelo baricentro G de um triângulo ABC e deixa o vértice
A de um lado e os vérticos B e C do outro. Prove que a soma das distâncias de B e C à reta r é igual à
distância de A a r .
Resolução

Exercício 2.3

Seja M o ponto médio de BC , P e Q os pés das perpendiculares baixadas de A e M à reta r , respectivamente.


Marcando os pontos R e S tais que R é o ponto médio de AG e S o pé da perpendicular baixada de R à reta
r.
Marcamos também os pontos T e V que são os pés das perpendiculares baixadas de B e C respectivamente
em relação à reta r . Temos que AM é mediana, G é baricentro, logo AG = 2G M , e como R é ponto médio
de AG, então AR = AR = AR.
Os ângulos S ĜR = M ĜQ pois são opostos pelos vértices e G ŜR = SM Q̂G = 90◦ , logo, pelo caso L A A o , temos
RGS ≡ GQM , portanto, RS = QM .
Em APG, RS = 21 AP e RS = QM + CV .
Queremos mostrar que AO = BT + CV .
No trapézio BCV T , MQ ∥ BT ∥ CV . Como M é o ponto médio de BC , pelo teorema da base média para
BT +CV
trapézios, temos QM = .
2
1
Como QM = 2 AP , temos:
1 1
AP = (BT +CV ) ⇒ AP = BT +CV
2 2

Exercício 2.5,página 68[JGB] Prove que, em todo triângulo, a soma dos comprimentos das medianas é
menor que 3/2 do perímetro e maior que 3/4 do perímetro do triângulo.

32
Solução: Tomemos um triângulo ABC qualquer, em que AB = c, BC = a e AC = b e ainda AE = ma é a
mediana relativa ao vértice A, B F = mb é a mediana relativa ao vértice B e C D = mc é mediana relativa ao
vértice C. Consideremos também 2p o perímetro do triângulo ABC, ou seja, 2p = a + b + c e o ponto G é o
encontro das três medianas.

Exercício 2.5

Primeiramente, vamos mostrar que ma + mb + mc < 2p e assim, concluir que ma + mb + mc < 3/2(2p).
Vamos prolongar AE até o ponto H de tal forma que AE = E F = ma.

Como C E = E B , segue que o 4AE B ∼


= 4H EC e 4AEC ∼
= 4H E B . Assim, AB = C H = c e AC = B H = b.
Usando a desigualdade triangular no 4AC H , temos:

AH < AC + CH ⇒ 2AE < b + c ⇒ 2ma < b + c.

Analogamente: 2mb < a + c e 2mc < a + b. Adicionando as três desigualdade, obteremos:

2ma + 2mb + 2mc < b + c + a + c + a + b ⇒ 2(ma + mb + mc) < 2 (a + b + c), dividindo ambos os membros
por 2, temos:

ma + mb + mc < a + b + c = 2p. Assim, 2p < 3/2(2p). Portanto, a soma dos comprimentos das medianas
é menor que 3/2 do perímetro do triângulo.

Agora, vamos mostar que a soma dos comprimentos das medianas é maior que 3/4 do perímetro (2p)
do triângulo. Para isso, voltaremos a figura inicial desse itém. Seja AB = c, BC = a, AC = b, AE = ma, B F =

33
mb, C D = mc. Observemos que

AG = 2/3(AE ) = 2/3 (ma)

BG = 2/3(B F ) = 2/3 (mb)

CG = 2/3(C D) = 2/3 (mc)

Aplicando a desigualdade triangular nos 4 AGB,4 AGC e 4 BGC, temos:

4AGB : AG + GB > AB , 4AGC : AG + CG > AC , 4BGC : BG + GC > BC ⇒ 2/3 (ma) + 2/3 (mb) > c, 2/3
(ma) + 2/3 (mc) > b e 2/3 (mb) + 2/3 (mc) > a. Somando as trê sinequações, temos:

4/3(ma) + 4/3(mb) + 4/3(mc) > a + b + c = 2p ⇒ ma + mb + mc < 3/4 (2p).


Portanto, com as duas demonstrações provamos que em todo triângulo, a soma dos comprimentos das
medianas é menor que 3/2 do perímetro e maior que 3/4 do perímetro do triângulo.

Exercício 2.7, página 68[IMB] Seja ABCD um trapézio de bases AB = 7cm e C D = 3cm e lados não
paralelos AD e BC. Se  = 43◦ e B̂ = 47◦ , calcule a distância entre os pontos médios das bases do trapézio.

Solução:

Exercício 2.7

No trapézio com AB = 7cm, C D = 3cm  = 43◦ e B̂ = 47◦ . M é o ponto médio de C D e N é o ponto


médio de AB . Traçando peralelas a AD e C B sendo estas passando por M encontramos o triângulo MEF
retângulo em M. Sendo AE = FB =1,5 logo EF= 4cm e MN é mediana do triângulo MEF, temos que M N =
2cm( Corolário 2.43). " A mediana relativa a hipotenusa de um triângulo retângulo é igual à metade da
mesma."

Exercício 2.9, página 68[RGA] As bases AB e CD de um trapézio têm comprimentos a e b respectivamente


com a > b. Se os lados não paralelos são AD e BC e ∠BC D = 2∠D AB , prove que BC = a − b.

Solução: Considere o trapézio ABC D com base maior AB = a e base C D = b


Seja o ângulo B CbD = 2D AB
b , ou seja, B CbD = 2x e D AB
b = x.
Traçando o segmento C E paralelo a AD temos um quadrilátero AEC D

34
Exercício 2.8

Como C D paralelo a AB e C D = AE temos que AEC D é um paralelogramo.


No triângulo BC E , temos:
D AB
b = E CbD = x, pois são ângulos correspondentes de um paralelogramo.
Então E CbB = x pois, B CbD = 2D AB
b .
b = x pois são ângulos alternos internos das paralelas AB e C D
E C EbB = D AB
Logo o triângulo BC E é isósceles de bases C E assim B E = BC .
Como DC = AE e AB = AE + B E
Então B E = AB - AE = a − b
Logo BC = a − b

Exercício 2.12, página 69:[NLD] Um triângulo ABC é retângulo em A, tal que BC = 2AB . Calcule as
medidas em graus de seus ângulos.

Solução: Inicialmente, a partir do vértice A, tracemos o segmento de reta AM, de tal forma que M seja
o ponto médio de BC.Nestas condições, e pelo fato de o triângulo dado ser retângulo em A, o segmento
AM representa a mediana relativa á hipotenusa BC. Dessa froma, sendo AB = a, e da nossa hipótese de
BC = 2AB , temos que a hipotenusa BC tem por medida BC = 2a. Sendo M o ponto médio de BC, segue
que B M = MC = a. Como em tod triângulo retângulo a mediana relativa à hipotenusa é igual a metade da
hipotenusa , segue que AM = BC
2 = a.Observa-se, assim que o triângulo ABM é equilátero, de medida AB =
AM = B M = a. Nesse contexto, cada um de seus ângulos mede 60◦ , ou seja, A BbM = B M b = 60◦ .
cA = B AM
Como B AM
b +C AM b = 90◦ e B AM
b = 60◦ , segue que C AM
b = 30◦ . Por outro lado,os ângulos C M
cA e B McA são
cA = 60◦ ,segue imediatamente que seu suplemento C M
suplementares. Como B M cA = 120◦ . Por fim , como
a soma doa ãngulos internos de um triângulo é igual a 180◦ , temos que C M b + Cb = 180◦ , donde
cA + C AM
segue que Cb = 180◦ − 120◦ − 30◦ = 30◦ . Conclusão: Ab = 90◦ , Bb = 60◦ e Cb = 30◦ .

Exercício 2.13, página 69[TCF] Em um triângulo ABC , sejam M o ponto médio do lado BC e Hb e Hc ,
respectivamente, os pés das alturas relativas a AC e AB . Prove que o triângulo M Hb H c é isósceles.

Solução

35
Exercício 2.13

Considere o triangulo BC Hc temos que Hc M é a mediana relativa a hipotenusa de um triângulo retângulo


portanto, pelo Corolário2.43 Hc M é igual a metade de BC . Agora, considerando o triângulo BC Hb pelo
mesmo corolário temos que M Hb é igual a metade de BC .
Concluimos então que M Hc = M Hb e sendo assim o triângulo M Hb Hc é isósceles.
.

36
Solução dos exercícios do capítulo 3, página 77

Exercício 3.1, página 77[AJS] Construa um círculo de raio dado r , que passe por dois pontos dados A e B .
Sob que condições há solução? (Sugestão: o centro do círculo desejado deve distar r do ponto A e pertencer
à mediatriz do segmento AB . Mostre que há uma solução se AB = 2r e duas soluções se AB < 2r .)

Solução: Descrição dos passos:

1. Tracemos uma reta s que passa por A e B .

2. Com centro em A e abertura maior 12 AB , traçamos um círculo.

3. Com centro em B e mesma altura traçamos outro círculo.

4. Marcamos C e D pontos de intersecção dos círculos.

5. Traçamos o segmento C D.

6. Seja E o ponto de intersecção de C D e AB .

Conclusão: i) Se o raio r dado é tal que r = AE , temos AB = 2r e portanto, temos uma única solução, centro
−→
em E . ii) Se r > AE , temos duas soluções: uma com centro na origem da semirreta EC e outra na origem
−−→
da semirreta E D. iii) Se r < AE , não há solução.

Exercício 3.1

Exercício 3.2,página 77[CA] Identifique e construa, com régua e compasso, o LG do vértice A de um


segmento AB , conhecida a posição do vértice B e o comprimento c de AB .

Solução:
1. Com a régua construímos a reta r tal que B ∈ r .
2. Com o compasso, façamos uma abertura c do comprimento dado. Fixamos a ponta seca sobre B e
façamos o círculo de centro B e raio c.
3. O lugar geométrico LG do ponto A é o conjunto de todos os pontos do plano que estão a igual distância
c do ponto B conhecido. Segue que compreende ser a circunferência de centro B e raio B A = c.

37
Exercício 3.2

Exercício 3.3, página 77[AA] Construa com régua e compasso um triângulo ABC , conhecidos os comprimentos
c do lado AB , a do lado BC e a medida α do ângulo ∠B AC .

Solução:

1. Sobre uma reta r traçamos um segmento AB de comprimento c;

2. Em A, construa o ângulo ∠B AC 0 = α; encontrando a reta s

3. Centro em B e raio igual ao segmento BC = a que corta a reta s no ponto C

4, O triângulo ABC é o triângulo pedido

Exercício 3.5 , página 77[CAR] Identifique e construa com régua e compasso o LG dos pontos do plano
equidistantes deduas retas paralelas dadas r e s.

38
Solução: Sejam as retas r e s paralelas. O lugar geométrico (LG) dos pontos do plano equidistantes das
retas r e s é toda reta t do plano, tal que d (C , r ) = d (C , s) onde C ∈ t .
Para encontrar o LG dos pontos equidistantes das retas r e s, marca-se dois pontos distintos, A e B, sobre
a reta r, em seguida, obtenha o ponto médio M do segmento AB . Em seguida, faz-se a mediatriz passando
por M e pelo ponto N ∈ s, que é a interseção da mediatriz com s. Marque o ponto C que é o ponto médio
do segmento M N . Trace a paralela a r, passando por C, e obtenha reta t, que representa o LG dos pontos
equidistantes das retas r e s.

Exercício 3.5

Exercício 3.6, página 77 [LOC] Construa com régua e compasso, o triângulo ABC, conhecidos os comprimentos
AB = c, BC = a e h a da altura baixada a partir de A

Solução Descrição dos passos:

1. Trace uma reta r e sobre ela construa o segmento BC = a

2. Construa uma reta s paralela a r com distância h a de r em um dos semiplanos determinados por r .

3. Construa uma reta t paralela a r com distância h a de r no outro semiplano determinado por r .

4. Fixe o compasso em B com abertura igual a AB = c marcando assim os pontos A 1 , A 2 ∈ s e os pontos


A3, A4 ∈ t .

5. Note que temos quatro soluções que satisfazem o lugar geomêtrico descrito no problema, são eles os
∆A 1 BC , ∆A 2 BC , ∆A 3 BC e ∆A 4 BC ,

39
Exercício 3.8, página 77[NLD] São dados, no plano, uma reta r, um ponto A 6∈ r e dois segmentos, de
comprimentos a e b. Construa, com régua e compasso, todos os pontos B do plano tais que AB = a e
d (B, r ) = b. Sob que condições sobre a e b há solução?

Solução: São dados, no plano, uma reta r, um ponto A 6∈ r e dois segmentos de comprimento a e b. i) O
lugar geométrico dos pontos B do plano tais que AB = a é um círculo de centro A e raio a.
ii) O lugar geométrico dos pontos B tais que d (B, r ) = b é a união das retas s e s’, paralelas a r e situadas ,
cada uma, à distância b de r.
Construção da figura:
1. Construir as reta s e s’, paralelas a r e situadas, cada uma a uma distância b de r.
2.Com o compasso centrado num ponto P que dista b unidadesde r, trace um circulo α, que intersecta a
reta r nos pontos distintos Q e R.
3.Ainda com o compasso centrado em P, trace o círculo β de raio igual a QR.
4.Com o compasso centrado em R, trace o círculo γ de raio igual ao raio de α.
5. Marque o ponto S de interseção de β e γ, situado no mesmo semiplano que P em relação à reta r.
Pela proposição anterior, PQRS é um paralelogramo; portanto , a reta PS é paralela à reta r.
6. Tomando o ponto A a uma distância a da reta r e 0 < b ≤ 2a, teremos solução. Veja a figura:

40
Exercício 3.9, página 77[EVO] Construa o triângulo ABC , conhecendo os comprimentos a do lado BC,
h a da altura relativa a BC e a medida α do ângulo ∠ A.

Solução:

Exercício 3.9

DESCRIÇÃO DOS PASSOS

1 Trace a reta r e marque sobre ela o ponto B.

2 Com o compasso centrado em B e abertura BC , transporte o segmento BC para a reta r.

3 Com o compasso centrado no vértice do ângulo dado, trace um arco de círculo de raio arbitrário R,
marcando pontos P e Q sobre os lados do mesmo.

4 Com o compasso centrado em B, trace outro arco de círculo de mesmo raio R, marcando o ponto Q’ na
interseção com r.

5 Com o compasso centrado em Q’, trace outro arco de círculo de raio PQ, marcando o ponto P’ na
interseção com o arco descrito no passo 4.

6 Trace a reta t passando por B e P’. O ângulo C B̂ P 0 é igual ao ângulo dados.

7 Obtenha a mediatriz de BC e marque o ponto M, interseção da mediatriz m com a reta r.

8 Marque o ponto X ∈ r, à direita de C.

9 Obtenha a mediatriz de C X e marque o ponto N, interseção da mediatriz n com a reta r.

10 Com o compasso centrado em M e abertura h BC , descreva um arco de círculo que intersecte a mediatriz
m no mesmo semiplano do ângulo C B̂ P 0 . Marque o ponto M’ na interseção.

41
11 Com o compasso centrado em N e abertura h BC , descreva um arco de círculo que intersecte a mediatriz
n no mesmo semiplano do ângulo C B̂ P 0 . Marque o ponto N’ na interseção.

12 Trace a reta s passando por M’ e N’. A reta s é paralela à r.

13 Trace a reta p, perpendicular a reta t, passando por B.

14 Marque o ponto O, interseção da reta p com a mediatriz m.

15 Com o compasso centrado em O e abertura OB , descreva o arco que passe por B e C e intersecte a reta
s. Marque o ponto A na interseção. O arco descrito é o arco capaz de ∠ A sobre BC .

16 Trace os segmentos AB e AC .
O triângulo construído ABC é o pedido no exercício.

Exercício 3.10, página 77[PSM] São, dados no plano uma reta r e um ponto A, com A 6∈ r . O ponto B varia
em r.Encontre com justificativa, o LG descrito pelo ponto médio do segmento AB.

Solução:

Exercício 3.10

Dado a variação de B na reta r,podemos ter os pontos B, B 10 , B 0 . Se M é o ponto médio de AB e M 10 o


ponto médio de AB 10 e M 0 o ponto médio de AB’observamos M M 0 é a base média do 4AB B 0 , logo M M 0 é
paralelo a B B 0 e consequentemente paralelo a r que contém os pontos B, B 10 , B 0 . Se M M 0 é paralelo a r, M 10
também pertence a M M 0 ou seja M, M 1 e M 0 são colineares.
Portanto variando a posição de B, obtemos uma reta paralela a r que será a reta s, e será o LG dos pontos
médios do segmento AB

Exercício 3.11, página 78[JCR] Em uma folha de papel está desenhado um círculo α, mas seu centro não
está marcado. Mostre como obter a posição do mesmo com régua e compasso.

Solução:

42
Exercício 3.11

Sobre o círculo α traçamos duas cordas distintas AB e BC .


Com o auxílio da régua e do compasso traçamos a mediatriz de AB e a mediatriz de BC O ponto de
interseção das duas mediatrizes, que chamaremos de O, é o centro da círculo α.
Como o ponto O pertence a mediatriz de AB e a mediatriz de BC , o ponto O é o LG que equidista dos
pontos A, B e C .

Exercício 3.13, página 78[JSO] Construa o triângulo ABC, conhecendo o semiperímetro p do mesmo e as
medidas β e γ dos ângulos B̂ e Ĉ , respectivamente.

1º Trace um segmento de reta medindo 2P e chame as extremidades de B 0 e C 0 ;

α β
2º Divida os ângulos α e β pela metade e transporte 2
e 2 para B 0 e C 0 respectivamente;
−−→ −−→ −−→ −−→
3º Trace semirretas B X e C Y ambos do mesmo lado do semiplano e chame de {A} = B X ∩ C Y ;
β
4º Transporte o ângulo 2 para o ponto A formando ângulo com o segmento BC e construa o segmento
AB onde B é o ponto de intersecção da semirreta com o segmento de reta B 0C 0 . Faça o mesmo com
α
o ângulo 2
em relação ao segmento AC 0 e chame o ponto de C ;
Temos que:
AB = B 0 B e AC = C 0C como B 0 B + BC + C 0C = 2P (Perímetro) e pelo teorema do ângulo externo
b = β e ACbB = α
A BC

43
Soluções do capítulo 3, página 84

Exercício 3.1, página 84[VT] De um triângulo ABC , conhecemos as posições dos vértices B e C e do
circuncentro o. Explique porque a posição do vértice A não fica determinada.

Solução: Tracemos a reta s suporte de BC e o círculo Γ de centro O e raio OB=OC, pois o circuncentro O é
o centro da circunferência circunscrita do triângulo ABC. Observe a figura.

Assim, o vértice A pode pertencer a qualquer ponto do círculo deiferente de B e C, onde A ∈ Γ.

Exercício 3.3, página 84[JGB] De um triângulo ABC, conhecendo as posições dos vértices B e C e do
ortocentro H. Construa, com régua e compasso, o vértice A.

Solução: Construção:

Marque dois pontos B e C não coincidentes e trace o segmento BC . Depois marque um ponto H não
pertencente a BC . Trace uma reta r perpendicular a BC e que passe por H. Trace o segmento B H e trace a

44
sua perpendicular s passando por C. Marque o ponto A na intersecção das retas r e s e ligue os pontos A, B
e C, encontrando assim, o 4ABC procurado.

Exercício 3.5, página 84[TCF] Seja ABC um triângulo de ortocentro H, incentro I e circuncentro O. Mostre
que ABC é equilátero se, e só se, dois quaisquer dos pontos H, I e O coincidirem.

Solução: Se ABC é equilátero ⇔ H, I, O coincidem.


Dado o triângulo ABC equilátero abaixo, temos:

Sabemos que Ab = Bb = Cb = 60◦

b = 30◦ , MCb A = 60◦ , logo, AMC


Em AMC, M AC c = 90◦ .
b = 30◦ , BCbN = 60◦ , logo, BNC
Em BNC, NBC b = 90◦ .

Concluímos que,AM , B N , CO são as alturas, logo I = H. Por outro lado, nos triângulos:

b = IBb A = 30◦ , assim, AIB é isósceles, logo IA = IB.


AIB, temos I AB

BIC, temos IBbC = ICbB = 30◦ , assim, AIB ≡ BIC, pelo caso ALA, temos IA = IB = IC, logo I = H = O.

Se H, I, O coincidem, então ABC é equilátero.

45
Soluções do capítulo 3, página 97

Exercício 3.1,página 97[ACR] Dados no plano um círculo Γ e um ponto P sobre o mesmo, mostre que a
reta tangente a Γ em P é única.

Solução: Cada ponto da cincunferência é único quando uma reta s tangente passa sobre ele. Para cada
reta tangente passando por uma circunferência gera um ponto um ângulo de 90◦ com raio da circunferência,
logo cada tangente gera um único ângulo de 90◦ . Como QO > PO = r e o ângulo OPQ = 90◦ e consequentemente
é o maior ângulo do triângulo POQ portanto Q ∈ Γ e assim P é o único ponto comum a reta s e a circunferência
Γ.

Exercício 3.1

Exercício 3.5,página 97[TCF] Temos, no plano, duas retas concorrentes r e s. Dado um número real R
> 0, construa todos os circulos de raio R, tangentes simultaneamente a r e a s.

Solução: Trace as bissetrizes b 1 e b 2 dos ângulos formados pelas retas r e s.


Trace retas perpendiculares em relacao a r e a s, nos quatro semiplanos determinados por r e s.
Marque sobre as retas perpendiculares a distancia R da reta r e da reta s.
Trace a reta paralela r 1 em relacao a reta r .
A intersecao de r 1 e b 2 é o ponto O 2 .
Proceda analogamente, obtendo O 1 ,O 3 eO 4 .
Logo, temos quatro solucoes.

Exercício 3.5

46
Exercício 3.9, página 98[AJS] São dados no plano, um círculo Γ de centro O e um ponto A ∈ Γ. Identifique
e construa, com regua e compasso, o LG dos centros dos círculos tangentes a Γ em A. (Sugestão: seja Γ0 um
←→
círculo de centro O 0 e raio R 0 . Se O 0 ∈ AO A e R 0 = AO 0 . mostre que Γ0 tangência Γ em A.)

Solução: Descrição dos passos:


−−→
1. Trace a semirreta O A.

2. Marque o ponto O 0 sobre a semirreta exterior à Γ e construa o círculo Γ0 de centro O 0 e raio O 0 A.

3. Como O A + AO 0 = OO 0 , Γ0 tangencia Γ em A. Logo, o LG é o conjunto de pontos que pertencem ao


segmento OO 0 exceto o ponto A.

Exercício 3.9

Exercício 3.11, página 97[AA] Sejam ABC D um quadrado de lado α e Γ o círculo de centro A e raio α.
Marcamos pontos M e N , respectivamente sobre BC e C D, tais que M N tangencia Γ. Quais os possíveis
valores do ângulo M ÂN ?

Solução: Seja ABCD um quadrado de lado a e seja δ a circunfêrencia de centro A e raio a.


Marcamos os pontos M e N sobre BC e C D de forma queM N tangencia δ. Consideremos T o ponto de
tangência de MN sobre δ, assim M N ⊥AT .
Observe que os vértice do quadrado, B e D também são pontos de tangências de δ, então B̂ ≡ D̂ = 90◦ .
Pela Proposição 2 (Os segmentos das tangentes traçadas por um ponto exterior a uma circunfêrencia são
iguais) temos que D N ≡ N T e T M ≡ M B , pois M e N são pontos exteriores e D, T e B pontos de tangência.
Analisando, primeiramente o ∆AT M e o ∆AT N : Como AD ≡ AT = a, D N ≡ N T e AN é lado comum, pelo
caso de congruência LLL, temos que ∆AD N e o ∆AT N . Daí, D ÂN = N ÂT = α1 .
Agora analisando o ∆AT M e o ∆AB M :
Como AB ≡ AT = a, B M ≡ M T e AM é lado comum, pelo caso de congruência LLL, temos que ∆AT M e o
∆AB M . Daí, T ÂM = M ÂB = β2 .
Sabemos que B ÂD = 90◦ 3 , por ser vértice do quadrado e
B ÂD = D ÂN + N ÂT + T ÂM + M ÂB
Por 1, 2 e 3, temos que
90◦ = α + α + β + β ⇒ 2α + 2β = 90◦ ⇒ α + β = 45◦

47
Mas, M ÂN = N ÂT + T ÂM ⇒ M ÂN = 45◦ logo, M ÂN = 45◦ .

Exercício 3.12. página 99[RCP] As retas r e s são concorrentes em A e tangentes a um círculo Γ, de centro
O.
←→ ˆ = 30ř,
Pontos P ∈ r e Q ∈ s são tais que PQ tangencia Γ e deixa A e O em semiplanos opostos. Se P AQ
ˆ
calcule POQ.

Solução:

Sabendo que r e s são concorrentes em A e que tangenciam a circunferência zG amma em B e C , e que


←→
PQ também tangência Γ em r , tem-se:
ˆ = 2POR
1. BOR ˆ e COR ˆ = 2QOR.ˆ
2. Considerando o quadrilátero ABOC , temos:
ˆ + OBˆ A + B AC
BOC ˆ + ACOˆ = 360ř −→ BOC
ˆ + 90ř + 30ř + 90ř = 360ř −→ BOC
ˆ = 150ř.
ˆ = BOR
3. De 1 e sabendo que BOC ˆ + COR,
ˆ temos:
ˆ = 2QOR
BOC ˆ −→ POR ˆ = 150ř . Logo, POQ
ˆ + QOR ˆ = 75ž.
2

Exercício 3.13, página 98[CAR] Dois círculos Σ e Γ se intersectam em dois pontos distintos A e B . Escolhemos
←→
X ∈ Σ e Y ∈ Γ tais que A ∈ X Y . Prove que a medida do ângulo ∠ X B Y independe da direção da reta X Y .

Solução:

48
Exercício 3.13

Sabe-se que a soma dos ângulos internos de um triângulo é igual a 180◦ . Logo:
∠ X B Y + ∠B X Y + ∠B Y X = 180◦ ⇒
⇒ ∠ X B Y = 180◦ − ∠B X Y − ∠B Y X .
Pelo Teorema do Ângulo Inscrito, tem-se que ∠B Y X é ângulo inscrito de Σ e ∠B X Y é ângulo inscrito de Γ.
Como ∠B Y X e ∠B X Y são opostos ao segmento AB, eles independem da direção da reta XY.

Exercício 3.14, página 98:[SLS] As cordas AB e CD de um círculo Γ são perpendiculares em E, um ponto


situado no interior do círculo.A reta perpendicular a AC por E intersecta o segmento BD em F. Prove que F
é o ponto médio de BD.

Solução:

Pelo Teorema do ângulo inscrito, temos que: B DC


b = B AC
b , pois são ângulos inscritos que corresponde
ao mesmo arco AB
” . Pelo teorema da soma dos ângulos internos: Em DEB, temos : Db +90◦ + Bb = 180◦ ⇒ D
b+
Bb = 90◦ (1). Em AEP , temos : Ab +90◦ + Eb = 180◦ ⇒ Ab + Eb = 90◦ (2). Fazendo (1) - (2) , temos : D
b + Bb − Ab − Eb = 0
(mas D b = A)b ⇒ Bb = Eb. Logo, o 4E F B é isósceles de base E B . Analogamente, Bb = Eb. Logo , o 4DF E é
isósceles de base E D. Logo, FD = FB e F é o ponto médio de B D.

49
Exercício 3.17, página 99[EVO] Analise a construção dos arcos capazes de um ângulo α sobre o segmento
AB, quando 90◦ < α < 180◦ .

Solução: DESCRIÇÃO DOS PASSOS

1 Trace a reta r e marque sobre ela os pontos A e B.

2 Transporte o ângulo α para r, com vértice em B e lados AB e BA’.


(Se necessário, consulte os passos de 3 a 6, da solução do exercício 3.9 da Pág. 77)

3 Trace a reta p, perpendicular ao segmento B A 0 , passando por B.

4 Obtenha a mediatriz de AB e marque o ponto M, interseção da mediatriz m com a reta r.

5 Marque o ponto O, interseção da reta p com a mediatriz m.


O ângulo M ÔB = 180◦ − α.

6 Com o compasso centrado em O e abertura OB , descreva o arco que passe por A e B.


Marque o ponto C no arco.
O arco descrito é o arco capaz de α sobre AB . Então, se C ∈ arco AB , ∠ AC B = α.

Exercício 3.18,página 99[IMB] Construa o triâgulo ABC,conhecendo os comprimentos a do lado BC, h a


da altura relativa a BC e a medida α do ângulo ∠ A .

Solução: Segue a figura abaixo:

Exercício 3.18

Descrição dos passos:


1.Trace a reta r marcando o ponto B sobre a reta r,com altura A e o centrado em B, construindo o lado BC.
2.Trace a reta s mediatriz de BC e marque sobre s o ponto M à distância h a de r.
3. Tracea paralela referente a BC passando pelo ponto M.
4. Construa o arco capaz de sobre BC.

50
Exercício 3.19, página 99[JCR] Sejam ABC uma triângulo e P e M , respectivamente, os pés da bissetriz
interna e da mediana relativa ao lado BC . Se P e M coincidirem, prove que ABC é isósceles de base BC .

Solução:

Exercício 3.19

Como a medida relativa ao lado BC e a bissetriz do ângulo Ab coincidem os pontos P e M são coincidentes.
α
Seja α a medida do ângulo A,
b temos que B AMb = C AM b = .
2
α
Consideremos o vértice A como a intersecção dos arcos capazes de construídos respectivamente sobre
2
segmentos de mesma medida B M = C M . Além disso possuem um segmento comum AM . Assim AB = AC .
Logo o triângulo ABC é isósceles de base BC .

Exercício 3.21, página 99[JSO] De um triângulo ABC, conhecemos a posição dos vértices B e C, a medida
←→
α do ângulo B ÂC e o semiplano β, dentre os determinados pela reta BC , no qual está situado o vértice
A. Quando A descreve o arco capaz de α sobre BC, situado no semiplano β, encontre o LG descrito pelo
incentro I de ABC.

Solução: Por construção encontramos o arco de circunferência no plano α como mostrado na figura.

Exercício 3.21

51
Exercício 3.23, página 99[JEO] Dois círculos Γ1 (O 1 ; R 1 ) e Γ2 (O 2 ; R 2 ) são secantes, intersectando nos pontos
A e B . Dado um segmento de comprimento l , explique como traçar, com régua e compasso, uma reta
passando por A, (dita sec ant e aos círculos), intersectando Γ1 e Γ2 respectivamente em X e em Y (com
X , Y 6= A), e tal que X Y = l . Explique sob que condições há solução.

Solução:

Exercício 3.23

1◦ : Traçar um segmento ligando os centros O 1 e O 2 .


2◦ : Encontrar o ponto médio M de O 1O 2 , centrar o compasso em M , abrir até O 1 e traçar uma circunferência
Γ3 .
3◦ : Encontrar o ponto médio do segmento l , abrir o compasso com uma medida igual à metade do segmento
l.
4◦ : Centrar o compasso em O 2 e traçar um arco que corte a circunferência Γ3 .
5◦ : Traçar um segmento que ligue O 2 ao ponto de cruzamento com Γ3 , chamaremos de P .
6◦ : Trace uma reta paralela à O 2 P passando por A.
7◦ : Marque os pontos X e Y .
8◦ : X Y = l .

Como é necessário que o arco com a metade da medida de l cruze o círculo Γ3 , ele precisará ser menor
l
que o comprimento entre O 1O 2 , ou seja O 1O 2 ≥ .
2

Exercício 3.28, página 100[JGB] São dados um círculo Γ e os pontos A, B e C sobre o mesmo, tais que AB
> AC . Marcamos o ponto médio M do arco BC que contém A, bem como o ponto N, pé da perpendicular
baixada de M ao segmento AB. Prove que B N = AN + AC .

−→
Solução: Marcaremos o ponto A’ ∈ B A/AB, tal que A A 0 = AC (1). Agora, pelo teorema do ângulo externo,
A 0 C = 1 B AC
temos que Bc cA’ é a medida do arco capaz sobre A 0 B , temos que
b e pelo exemplo 10, como BM
2

52
M B = M A 0 , logo Γ2 pelo ponto A’. Como M N ⊥ A 0 B , temos que o triângulo A’MB é isósceles de base A 0 B ,
segue que N é ponto médio de A 0 B , assim B N = A 0 N . (2)

Logo, B N + N A 0 = B N + AN + A A 0 . Substituindo (1) e (2), temos:


B N + B N = B N + AN + AC ⇒ B N = AN + AC .

53
Soluções dos exercícios do capítulo 3, página 109

Exercício 3.1, página 109[RGA] Construa o triângulo ABC conhecendo os comprimentos do raio r do
Círculo circunscrito e a e b dos lados BC e AC respectivamente

Solução: Descrição dos passos:


1) trace o círculo Γ no plano com centro O e raio R;
2) marque o ponto C no círculo Γ e em seguida com centro em C e abertura BC = a marque o ponto B
sobre Γ;
3) com centro em C e abertura AC = b marque o ponto A sobre Γ;
4) trace o lado AB do triângulo ABC circunscrito.

Exercício 3.1

Exercício 3.5, página 110[TCF] Seja ABC um triângulo de ortocentro H e circuncentro O. Prove que a
bissetriz interna relativa ao lado BC tambem bissecta o ângulo H AO
b

Soluçao: Considerando o caso em que ABC e acutangulo, sendo os demais casos totalmente análogos.
Temos O AC b ) = 1 (180 − 2Bb) = 90 − Bb = B AI
b = 1 (180 − AOC b . Agora, sendo I o incentro de ABC , temos
2 2
H AI
b = B AI
b − B AH
b = C AI
b − O AC
b = O AI
b .

Exercício 3.5

Exercício 3.9, página 100[AJS] Seja ABC um triângulo retângulo em A e H o pé da altura relativa à
hipotenusa BC . Sejam, ainda, I 1 e I 2 os incentros dos triângulos AB H e AC H . Prove que A é o ex-incentro
do triângulo I 1 H I 2 relativo ao lado I 1 I 2 . (Sugestão: use o resultado do problema anterior.)

54
Solução: Segundo o corolário 21, em todo triângulo, a bissetriz interna relativa a um vértice concorre
com as bissetrizes externas relativas aos dois outros dois vértices no ex-incentro. Em nosso problema,
H é o vértice, H A é bissetriz interna de I 1 H
b I 2 e AI 1 e AI 2 são bissetrizes externas dos vértices I 1 e I 2 do
nosso triângulo, segue que AI 1 = AI 2 . Logo, a intersecção de dois segmentos é o ponto A. Portanto, A é o
ex-incentro do triângulo I 1 H I 2 relativo ao lado I 1 I 2 .

Exercício 3.11, página 110[AJS] Em um triângulo ABC , marcamos os pontos Q e R, de tangência do lado
BC respectivamente com o círculo inscrito em ABC e ex-inscrito a ABC em relação ao lado BC . Se P é o
pé da perpendicular baixada de B à bissetriz interna de ABC relativa ao vértice A, mostre que Q P̂ R = 90◦ .

Exercício 3.11

Solução: Marcaremos o ponto médio M relativo ao lado BC e o ponto P que é o pé da perpendicular


baixada de B na bissetriz interna de ABC relativa ao vértice A.
1
Mostraremos que P M = (b − c), com b = AC e c = AB .
2
Marquemos inicialmente o ponto Q de intersecção de B~P com AC .
1) Mostraremos inicialmente que P é o ponto médio de BQ.
Temos que:
i) AP é bissetriz de Â, logo, B ÂP = Q ÂP .
ii) BQ⊥AP , então B P̂ A = A P̂Q = 90◦
iii) AP é o lado comum de AP B e APQ

55
Exercício 3.11

Pelo caso ALA, temos que AP B e APQ, logo B P = PQ e P é o ponto médio do lado PQ. (c.q.d)

1
2) Mostraremos agora que P M = (b − c)
2

No triângulo BQC , temos que P é o ponto médio de BQ e M é o ponto médio de BC , logo, pelo teorema
1 1
da base média do triângulo, P M = QC = (AC − AB ).
2 2
Como AQ = AB , segue que
1 1
P M = (AC − AB ) = (b − c). (c.q.d)
2 2

Tracemos agora o círculo inscrito a ABC e o círculo ex-inscrito ao triângulo ABC pelo lado BC e marquemos
seus pontos de intersecção Q e R, respectivamente, com o lado BC.

Queremos mostrar inicialmente que M também é o ponto médio de QR. Para isso, vamos mostrar que
BQ = RC .

3) Marquemos os pontos X e Y que são as intersecções dos prolongamentos dos lados AB e AC, respectivamente,
com o círculo ex-inscrito e os pontos S e T que são as intersecções dos lados AB e AC com o círculo inscrito.

Sabemos que
AX = AY =⇒ AS +SB +B X = AT +T C +C Y =⇒ BQ +BQ = QC +C R =⇒ BQ +BQ +QR = QR +C R +C R =⇒
2BQ = 2C R =⇒ BQ = C R. (c.q.d)

Logo, M é o ponto médio de QR e PK é uma mediana de PQR.

4) Mostraremos agora que Q P̂ R = 90◦ .


Tomemos AT = AS = x, B S = BQ = y e CQ = C T = z.
Pelo que já foi mostrado na proposição 22, temos que
x = p −a

56
y = p −b
z = p − c, p = semiperímetro
logo,
QR = QC − RC
QR = z − BQ
QR = z − y
QR = p − c − (p − b)
QR = b − c

1 1
Sabemos que PM é a mediana relativa a QR. Como P M = (b − c) e QR = b − c, temos que P M = QR,
2 2
logo, pelo colorário 16 da unidade 5 que afirma que a mediana relativa a hipotenusa de um triângulo
retângulo é igual à metade da mesma, temos que Q P̂ R = 90◦ .

57
Exercícios do capítulo 3. página 117

Exercício 3.1, página 117[CAR] Sobre cada lado do triângulo ABC , construímos um círculo tendo o
referido lado por diâmetro. Prove que tais círculos se intersectam dois a dois em seis pontos, três dos
quais são os pés das alturas de ABC .

Solução:

Exercício 3.1

Percebe-se que na figura, que três pontos de intersecção são os vértices do triângulo ABC. O que queremos
mostrar é que BC , EC eAF são alturas do triângulo ABC.
Percebe-se que:

1. BC é o diâmetro do círculo σ1, e como ∠B DC está inscrito em σ1, logo, ∠B DC = 90◦ , o que implica
que BC é altura do triângulo relativa ao lado AC.

2. AB é o diâmetro do círculo σ2, e como ∠ AF C está inscrito em σ2, logo, ∠ AF C = 90◦ , o que implica
que AF é altura do triângulo relativa ao lado BC.

3. AC é o diâmetro do círculo σ3, e como ∠ AEC está inscrito em σ3, logo, ∠ AEC = 90◦ , o que implica
que AE é altura do triângulo relativa ao lado AB.

Portanto, os círculos σ1, σ2, σ3 se intersectam dois a dois em seis pontos, sendo três deles os vértices
triângulo ABC e três sendo os pés das alturas de ABC.

Exercício 3.5, página 117[EVO] Dado um triângulo ABC com círculo circunscrito Γ, sejam P um ponto
situado sobre o arco AC de Γ que não contém o vértice B e D o pé da perpendicular baixada de P à reta
←→
suporte do lado BC . Se Q 6= P é o outro ponto de interseção da reta DP com o círculo Γ e r denota a reta de

58
←→
Simson-Wallace deP em relação a ABC , prove que r k AQ.

Solução:

Exercício 3.5

Observe que as retas i e f são paralelas, pois D ∈ i é o pé da perpendicular baixada de P à reta suporte
do lado BC e J ∈ f , mediatriz do lado BC. Além disso, os ângulos alternos internos D Q̂ A e L D̂Q são iguais.
←→
Então, r k AQ.

Exercício 3.7, página 117[JCR] Um polígono convexo é incritível se exitir um círculo passando por seus
vértices, dito o círculo circunscrito ao polígono. Prove que um polígono convexo é inscritível se, e só se, as
mediatrizes de seus lados concorrem em um único ponto.

Solução: Seja O o circuncentro de um polígono A 1 , A 2 , A 3 , . . . , A n−1 , A n . Como O é o ponto de interseção


das mediatrizes de A 1 A 2 , A 2 A 3 , . . ., A n−1 A n , A n A 1 , temos que O equidista dos vértices A 1 , A 2 , A 3 , . . . , A n−1 , A n
do polígono. Sendo r tal distância, segue-se que o círculo de centro O e raio r é circunscrito ao polígono
A 1 , A 2 , A 3 , . . . , A n−1 , A n .
Reciprocamente um polígono inscritível deve ter todos os seus vértices pertencentes ao círculo de centro
O.
Como O é o ponto equidistante de A 1 , A 2 , A 3 , . . . , A n−1 , A n , temos que O pertence as mediatrizes A 1 A 2 ,
A 2 A 3 , . . ., A n−1 A n , A n A 1 . Portanto o ponto O é a interseção dessas mediatrizes.

59
Exercício 3.9, página 118[JSO] Um polígono convexo é circunscritível se existir um círculo tangente a
seus lados, o qual é denominado um círculo inscrito no polígono. Prove que um polígono convexo é
circunscritível se, e somente se, as bissetrizes de seus angulos passarem todas por um mesmo ponto. Nesse
caso, conclua que há um único círculo inscrito no polígono.

Solução: ⇒
Se um polígono é circunscritível então existe um cículo Γ de centro O e tangente aos lados desse polígono.

Se A é um vétice desse polígono e os pontos B e C são os pés das perpendiculares baixadas a partir de
O sobre os lados Adjacentes a A.

Exercício 3.1

Temos que os triângulos O A e O AC são congruentes pelo casa LAL, logo AO é bbissetriz de C AB .

Como podemos refazer esse raciocínio para todos os vértices do polígono 2 a 2, temos que o encontro
dessas bissetrizes será sempre o pontoO.


Dado um polígono P n e um ponto denominado I , onde todas as bissetrizes dos vértices desse polígono se
encontram.

60
Exercício 3.9

Seja I E e I B os segmentos de reta perpendiculares,etão temos os triangulos congruentes AI B e AIC .


Como podemos refazer esse raciocínio para todos os lados do polígono 2 a 2, temos que todas as distancias
de I aos lados são iguais, portanto o polígono convexo é circunscritível.

Exercício 3.11, página 118[JEO] Se um hexágono convexo A 1 A 2 A 3 . . . A 6 é circunscritível, prove que A 1 A 2 +


A3 A4 + A5 A6 = A2 A3 + A4 A5 + A6 A1.

Solução:

61
Exercício 3.11

Dado o hexágono A 1 A 2 A 3 A 4 A 5 A 6 , se traçarmos um segmento A 1 A 4 dividimos o hexágono dado em


dois quadriláteros:
A1 A4 A5 A6e A1 A2 A3 A4

Como A 1 A 4 A 5 A 6 é circunscritível, temos que: A 1 A 4 + A 5 A 6 = A 1 A 6 + A 4 A 5 (1)


Como A 1 A 2 A 3 A 4 é circunscritível, temos que: A 1 A 2 + A 3 A 4 = A 1 A 4 + A 2 A 3 (2)
Somando (1) e (2) temos:

A1 A4 + A5 A6 + A1 A2 + A3 A4 = A1 A6 + A4 A5 + A1 A4 + A2 A3,

cancelando A 1 A 4 dos dois lados da igualdade, temos que:

A5 A6 + A1 A2 + A3 A4 = A1 A6 + A4 A5 + A2 A3

62
Solução dos exercícios do capítulo 4,página 127

Exercício 4.1, página 127[JGB] As retas r, s e t são paralelas, com s entre r e t. As transversais u e v
determinam, sobre r, s e t, pontos A, B, C e A’, B’ e C’, respectivamente, tais que AB = x + 2, BC = 2y, A 0 B 0 = y
e B 0C 0 = (x - 10)/2. sabendo que x + y = 18, calcule AB .

Solução:

Exercício 4.1

AB = x + 2

BC = 2y

A0B 0 = y

B 0C 0 = (x - 10)/2

x + y = 18 ⇒ y = 18 - x. Assim:

x−10
2y x−10
x+2 = 2
y ⇒ 2y2 = (x + 2) . 2

Usando y = 18 - x e fazendo todas as manipulações algébricas necessárias, temos:


136±52
3x2 - 136x + 1326 = 0 ⇒ x = 6 ⇒ x’ = 94/3 ou x" = 14.

Testando em y, temos:

Para x = 14 ⇒ y = 18 - 14 ⇒ y = 4.
Para x = 94/3 ⇒ y = -40/3 (não serve).

63
Logo, AB = x + 2 = 14 + 2 = 16 unidades de comprimento.

Exercício 4.3, página 127[JGB] Dados segmentos de comprimentos a e b, dizemos que um segmento de
comprimento x é a terceira proporcional de a e b (nessa ordem) se

a
b
= bx .

Solução: 1) Trace duas retas r e s, concorrentes em A;


2) Marque sobre r os segmentos AB = BC , tais que AB = a e BC = b;
3) Marque sobre s, o segmento B D, tal que B D = b;
4) Trace por C a paralela a B D a qual intercepta s no ponto E. Pelo teorema de Tales, temos:

a b
b = x ⇔ = b2 /a.


Exercício 4.4, página 117[AJS] Complete a prova do teorema da bissetriz.
Seja ABC um triângulo tal que AB 6= AC .

Exercício 4.4

BP BQ
a) Se P é o pé da bissetriz interna e Q é o pé da bissetriz externa relativa ao lado BC , então = = BA
PC QC AC
b) Sendo AB = c, AC = b e BC = a, temos
ac ab ac ab
B P = b+c , PC = b+c , B P = |b−c| e QC = |b−c|

64
Exercício 4.4b

BQ
BA
←→ ←→
Vamos provar a), mostraremos que =. Trace pelo ponto B , a reta B B 0 paralela à AQ e marque B 0
QCAC
ponto de intersecção com AC . b , obtemos: A BbB 0 = B AQ
Como Q A//B B 0 e AQ é bissetriz de B AX b = Q AX
b =
B Bb0 A.
Portanto, o triângulo AB B 0 é isósceles de base B B 0 , de maneira que B 0 A = B A. Das
←→ ←→0 ←→ ← → BQ AB 0
AQ e B B intersectadas pelas retas QC e AC , pelo teorema de Tales obtemos: = = BA.
QC AC AC
BP BA
Agora, mostraremos que = .
PC AC
←→0 ←→
Trace pelo ponto B , a reta B B paralela à AP e marque B 0 ponto de intersecção com AC . Como B B 0 //AP
e AP é bissetriz de B AC
b , obtemos: B APb = A BbB 0 = B Bb0 A.
←→ ← →
Portanto, o triângulo AB 0 B é isósceles de base B B 0 , de maneira que AB 0 = AB . Das paralelas B B 0 e AP
←→ ←→ 0
intersectadas pelas retas BC e B 0C , pelo teorema de Tales obtemos: B P = B A = B A .
PC AC AC

Exercício 4.4

Para provar o item b) é imediato de a).


BP BQ
Temos: = = BA.
PC QC AC
(
x c
x+y =a
Fazendo B P = x e PC = y, temos: x + y = a e y
= b
⇒ x
y
= bc
ab ab ab+ac−ab ac
x = a − y ⇒ (a − b)b = yc ⇒ ab − yb = yc ⇒ ab = yc + y 0 ⇒ y = b+c e x = a − b+c = b+c
= b+c

65
ac ab
Logo, x = b+c e y = b+c
(
z w = z +a
Fazendo BQ = z e QC = w, temos: w = z + a e w
= bc ⇒ z
w
= bc
ac ac ac+ab−ac ab
zb = wc ⇒ zb = (z + a)c ⇒ zb − zc = ac ⇒ z(b − c) = ac ⇒ z = |b−c| e w = b−c +a = b−c
= b−c
ac ab
Logo, z = |b−c| e w = b−c

Exercício 4.4

Exercício 4.5, página 127[RGA] Sejam ABC um triângulo e P e M respectivamente os pés da bissetriz
interna e da mediana relativa ao lado BC. Se P e M coincidirem prove que ABC isósceles de base BC.

Solução: Como M é coincidente com P, então pelo caso de congruência L.L.Ao. (lado, lado e lado oposto
ao vértice) o triângulo AB M é congruente ao triângulo C B M pois:
1o ) AM = BM, pois são mediana do triângulo ABC ;
2o ) BM é lado comum;
3o ) A BbM = C BbM pois são bissetriz do triângulo ABC ;
Logo AB = BC , ou seja, o triângulo ABC é isósceles de base BC.

Exercício 4.5

Exercício 4.6, página 128[AA] Em triângulos ABC , seja P o pé da bissetriz interna relativa a BC . Construa
o triângulo com régua e compasso, conhecendo os comprimentos P B , PC e AB .

Solução: Conhecendo os valores de P B = x 1 , PC =2 e AB = c, e sendo P o pé da bissetriz interna podemos


utilizar o teorema da bissetriz para determinar os lados AC = b e BC = a, logo temos que, se:

ac ab
BP = e BP = logo podemos concluir que:
b +c b +c

66
c.x 2
b= e a = x1 + x2
x1
Considerando os segmentos:
c.x 2
1) Construimos um Segmento AC =
x1
2) Centro em A e raio AB traçamos o arco m;
3) Centro em B e raio BC = x 1 + x 2 , traçamos outro arco n, encontrando o ponto B.

Exercício 4.7, página 128[ACR] Em um triâgulo ABC, sejam P o pé da bisstriz interna relativa ao vértice
A. Marcamos, respectivamente sobre AB e AC,pontos M e N tais que B M = B P e C N = C P. Prove que M N ∥
BC .

Solução: Dada a figura abaixo:

Exercício 4.7

BP
Pelo teorema de bissetriz, temos: CP
= AB
AC
BN AB
Como B M = B P e C N = C P . Assim C N = AC . Verificando o teorema de Tales se aplica no 4AM N e 4ABC
, assim poderemos provar que M N ∥ BC .
Aplicando o teorema de Tales, teremos :
AM
AB
= ANAC
. Como AN = AC − C N e AM = AB − B M , substituindo na razão acima teremos: AB −B M
AB
=
AC −C N
AC → BC M AB
N = AC . Logo pelo teorema de Tales concluímos que M N ∥ BC .

Exercício 4.8, página 128[CAR] Construa com régua e compasso o triângulo ABC , conhecendo os comprimentos
m a , m b e m c das medianas de ABC , respectivamente aos lados BC , AC e AB .

67
Solução: Sejam G o baricentro e M a , M b e M c os pontos médios dos lados BC , AC e AB , respectivamente.
−−−→
Marca-se o ponto P ∈ G M a , tal que G M a = M a P . Como as diagonais de GC P B se intersectam nos respectivos
pontos médios, o quadrilátero GC P B é um paralelogramo, e B P = GC . Pela Proposição 37, GP = 23 m a ,
GB = 23 m b , B P = 23 m c . Construindo segmentos de comprimentos 23 m a , 32 m b e 32 m c , encontra-se os pontos
A, B e C do triângulo ABC .

Exercício 4.8

68
Soluções do capítulo 4, página 137

Exercício 4.1, página 137[TCF] Prove que os conjuntos de condicoes elencados em cada uma das Proposicoes
7 e 8 sāo realmente suficientes para garantir semelhancas dos triangulos ABC e A 0 B 0C 0 ?

Solução: Proposicao 7: Sejam ABC e A 0 B 0C 0 tais que:


AB
= BC = k e Bb = Bb0
A0 B 0 B 0C 0
Entao ABC ∼ A 0 B 0C 0 com correspondencia de vertices A ←→ A 0 , B ←→ B 0 e C ←→ C 0 .
c0 e AC = k
A 0 , Cb = C
Em particular Ab = c
A 0C 0
Observe os triangulos:

Dado que AC = c, A 0 B 0 = kc, BC = a, B 0C 0 = ka e Bb = Bb0 = α. Queremos mostrar que ABC ∼ A 0 B 0C 0 .


Suponha, sem perda de generalidade, k > 1 e marque o ponto A 00 ∈ AB tal que B A 00 = B 0 A 0 .
Sendo C 00 a intersecao com o lado BC da reta que passa por A 00 e é paralela ao lado AC , segue pelo Teorema
de Tales que:
A 00 B BC 00
= = k1 , de maneira que A 00 B = 1
k
·AB = 1
k
·kc = c = A 0 B 0 . Trace agora a paralela a AB passando
AB BC
00
por C , a qual intersecta o lado AC no ponto D.

A 00C 00
Entao o quadrilatero A 00C 00 D A é um paralelogramo, novamente pelo Teorema de Tales temos: =
AC
BC 00
= k1 .
BC
1
Logo BC 00 = k
BC = B 0C 0 .
Portanto como A 00 B = A 0 B 0 e BC 00 = B 0C 0 e por hipotese Bb = Bb0 temos que pelo caso LLL os triangulos
A 00 BC 00 ≡ A 0 B 0C 0 .
AC
Logo temos: Ab = c A 0 , Cb = C
c0 e =k
A 0C 0

69
Proposicao 8:
AB AB
Sejam ABC e A 0 B 0C 0 triangulos, tais que Ab = c
A 0 e Bb = Bb0 . Entao ABC ∼ A 0 B 0C 0 . Em particular = =
A0 B 0 B 0C 0
AC c0 .
e Cb = C
C 0C 0

Suponha, sem perda de generalidade K > 1 e marque o ponto A" ∈ AB , tal que A A 00 = A 0 B 0 . Observe a
figura:

Sendo C " a interseçao com o lado BC da reta que passa por A" e e paralela ao lado BC , pelo paralelismo
de A"C " e BC temos:
A A"C " = A BC
b
Pelo caso de consequencia ALA em A A"C " e A 0 B 0C 0 temos que A A"C " = A 0 B 0C 0 . Assim C
c0 = Cb e pelo
AB AC BC
Teorema de Tales concluimos que = = =K
A0 B 0 A 0C 0 B 0C 0

Exercício 4.2, página 137[CA] Na figura abaixo, os três quadriláteros mostrados são quadrados e os pontos
X , Y e Z são colineares. Calcule, em centrímetros, a medida do lado do quadrado menor, sabendo que os
outros dois quadrados têm lados medindo 4 cm e 6 cm.

70
Solução:
1. Seja r a reta suporte das bases dos três quadrados tal que X 1 , Y1 , Z1 ∈ r . Trace por X , Y e Z , respectivamente,
as retas s, t e u paralelas à r .
←−→ ←−→ ←−→ ←−→
2. Temos que as retas X X 1 , Y Y1 e Z Z1 são perpendiculares à s. Em particular, {X 2 } = X X 1 ∩ t e {Y2 } =
←−→
Y Y1 ∩ u, sendo X 2 e Y2 o pé da perpendicular, respectivamente.
3. Dado que X , Y e Z são colineares, sejam os triângulos X X 2 Y e Y Y2 Z . Do paralelismo, temos que os
ângulos X Yb X 2 e Y Zb Y2 são correspondentes, ou seja, X Yb X 2 = Y Zb Y2 . Como já observado que X X
c2 Y =
c2 Z = 90◦ , segue pelo caso AA de semelhança de triângulos que X X 2 Y ∼ Y Y2 Z .
YY

4. Logo, pela semelhança de triângulos segue que

X X2 X2Y X X 1 −X 1 X 2 Y Y1 −Y1 Y2 2 4−Y2 Z


= ⇒ 4
= ⇒ 4
= ⇒
Y Y2 Y2 Z Y2 Z Y2 Z

71
⇒ 2Y2 Z = 16 − 4Y2 Z ⇒ 6Y2 Z = 16 ⇒ Y2 Z = 83

8
5. Portanto, a medida do lado do quadrado menor é 3 cm.

Exercício 4.3, página 138[WAM] Sejam os triângulos semelhantes abaixo:


sendo ma e m 0 a, ha e h 0 a, βa e β0 a, respectivamente os comprimentos das medianas, altura e bissetrizes
internas a A e A’.
Queremos provar que:

ma ha βa
= = =k
m0a h0a β0 a

Sabemos que

AB BC AC
= = =k
A0B 0 B 0C 0 A 0C 0

Vamos dividir o problema em três casos:

ma
i) Vamos mostrar inicialmente que =k
m0a
Sejam M e M’, os pontos médios dos lados BC e B 0 A 0 respectivamante, relativos aos vértices A e A’.

Em ABC , B M = C M = x, e em A 0 B 0C 0 = y,

Como ABC ∼ A 0 B 0 E 0 segue que:

AB BC B M + MC 2x x
= = = = =k
A0B 0 B 0C 0 B 0 M 0 +C 0 M 0 2y y
BM
assim, =k
B 0M 0

logo, pelo caso de semelhança,L AL, concluímos que AB M ∼ A 0 B 0 M 0 , daí

AB BM AM ma ma
= = = = k. Portanto, =k
A0B 0 B 0M 0 A0 M 0 m0a m0a
ha
ii) Vamos mostrar que =k
h0a
Sejam H e H’ os pés das alturas relativas aos vértices A e A’, respectivamente baixados em BC e B 0C 0
Em ABC temos que A Ĥ B = 90◦ e A 0 Ĥ 0 B 0 = α
Pelo caso de semelhança AA, concluímos que:
AH B ∼ A 0 H 0 B 0 .

72
AB BH AH ha ha
Daí, = = = = k. Portanto, =k
A0B 0 B 0H 0 A0 H 0 h0a h0a
AC
iii) Vamos mostrar que =k
A 0C 0
Sejam P e P’ os pontos de intersecção das bissetrizes relativas aos vértices A e A’, respectivamente baixados
em BC e B 0C 0
Em ABC temos que B ÂP = C ÂP = x e em A’B’C’ temos que B 0 Â 0 P 0 = C 0 Â 0 P 0 = y
Como a soma dos ângulos internos de qualquer triângulo é igual a 180◦ e B̂ = B̂ 0 e Ĉ = Cˆ 0 , então x = y
Pelo caso de semelhança AA, pois x = y e B̂ = B̂ 0 , concluímos que:

AP B ∼ A 0 P 0 B 0 .

AB BP AP βa βa
Daí, = = = = k. Portanto, =k
A0B 0 B 0P 0 A0P 0 βa 0 βa 0

Exercício 4.4, página 138[EVO] O triângulo ABC é retângulo em A e o ponto P ∈ BC é o pé da bissetriz


interna do ângulo ∠B AC . Calcule a distância de P ao lado AC em função de AB = c e AC = b.

Solução:

Exercício 4.4

Usando semelhança de triângulo e o fato de que ABC é retângulo em A,

PB PC
=
AB AC

PE PC CE
= = e
AB BC AC

73
bc
PE =
b +c

Exercício 4.5, página 138[AJS] Seja ABC um triângulo retângulo em A e tal que AB = 1. A bissetriz do
ângulo ∠B AC intersecta o lado BC em D. Sabendo que a reta que passa por D é perpendicular a AD e
intersecta o lado AC em seu ponto médio, calcule o comprimento do lado AC . (Sugestão: use o resultado
do problema anterior).

Solução: Observe o triângulo retângulo abaixo.


Temos que: AC = b, AB = c = 1, AM = C M , M é o ponto médio de AC . O triângulo AD M é isósceles, de
base AM . DL é mediana, bissetriz relativa a D. Assim, AL = LM e DQ⊥AB ⇒ AQ = DQ de onde concluímos
bc b
que AQDL é um quadrado. Daí, AC = 4d . Pela questão anterior temos, d = b+c , c = 1 ⇒ d = b+1 ⇒ d = 4d4d+1
⇒ 4d 2 + d = 4d ⇒ 4d 2 − 3d = 0 ⇒ d = 43 . Logo, AC = 4 · 34 = 3 cm.

Exercício 4.5

Exercício 4.6 página 138[JCR] Seja ABC D um paralelogramo de diagonais AC e B D e lados AB = 10cm,
AD = 24cm. Sejam, ainda E e F respectivamente os pés das perpendiculares baixadas desde A aos lados
BC e C D. Sabendo que AF = 20cm calcule o comprimento de AE .

Solução:

Exercício 4.6

Pelo caso LLL de semelhança de triângulos, temos que ADB ∼ C DB . Logo, B DC


b = D Bb A = x e A DB
b =
C BbD = y.
Assim, A DC
b = B Cb A = x + y.

74
Pelo caso de semelhança de triângulos AA, concluímos que F AD ∼ E AB . Portanto,

AD AF 24 20 20
= ⇒ = ⇒ AE = cm
AB AE 10 AE 3

Exercício 4.7, página 138[AA] Dois círculos, de raios r < R, são tangentes exteriormente em P . Sabendo
que tais círculos também tangenciam os lados de um ângulo de vértice A, calcule AP em termos de r e R.

Exercício 4.7

Solução: Queremos calcular AP , em funcão de r e R.


Pelo caso de semelhança ALA, temos AOB ∼ AO 0 B 0 .

OB AO
Daí, = mas,OB = r e O 0 B 0 = R e ainda O A 0 = AO + R + r logo,
O0B 0 AO 0
r AO
= =⇒ AOr + r R + r 2
R AO + R + r
2r R
=⇒ AP =
R −r

Exercício 4.8, página 138[JSO] Seja ABC um triângulo tal que BC = a, AB = c, e M , N e P pontos
respectivamente sobre AB , BC e AC , tais que AM N P é um losango.
a) Calcule, em termos de a, b e c, o comprimento do lado do losango.

Solução: Para calcular AP ,em função de a,b e c, seja dado o losango AM N P como P N é paralelo a
AB ,temos que A B̂C = P N̂C = α.
Assim os triângulos ABC e P NC são semelhantes pelo caso AA.

75
AB AC
=
AM ĹPC

c b bc
d = b−d ⇒ bd = bc − cd bd + cd = bc ⇒ d (b + c) = bc ⇒ d = b+c

Exercício 4.8

b) Mostre como construir com régua e compasso a posição do ponto M . SOLUÇÃO:

Com régua e compasso trace a bissetriz do ângulo Â, que vai intersectar olado BC no ponto N .
trace a mediatriz do segmento AN que irá intersectar o lado AC no ponto P e o lado AB no ponto M .

Exercício 4.9, página 138[CAR] Seja ABC um triângulo equilátero de lado a e M o ponto médio de AB.
←→
Escolhemos um ponto D sobre a reta BC , com C entre B e D, de modo que C D = a2 . Se AC ∈ D M = E ,
calcule AE em função de a.

Solução:

Exercício 4.9

Fazendo a paralela a AB passando por C e marque o ponto F na intersecção com B E . Como C F ∥ B A,


temos que ∠B M D = ∠C F D e, ∠M DB = ∠F DC . Logo, os triângulos CFD e BMD são semelhantes pelo caso
AA. Temos ainda que, ∠M AE e ∠F C E são alternos internos, logo, são iguais. E ∠ AE M = ∠C E F , pois são
opostos pelo vértice. Portanto, os triângulos CFE e AME são semelhantes.
Então:
CD CF FD C D.B M
BD = BM = MD ⇒ C F = BD (1)
CF FE
AM
=M E
= CAEE ⇒ C F = C EAE
.AM
(2)

76
Igualando as equações 1 e 2, temos:
C D.B M
BD = C EAE
.AM

Como AM = B M :
CD
BD
= CAEE ⇒ 3a/2
a/2
= CAEE ⇒ AE = 3C E
Como AC = AE + EC , temos:
AC = AE + AE
3
⇒ a = 4AE
3
⇒ AE = 3a
4
.

Exercício 4.10, página 138[JEO] Em um trapézio ABC D de bases AB = a e C D = b, os lados não paralelos
são AD e BC . Pelo Ponto de concursos P das diagonais de ABC D, traçamos o segmento M N paralelos às
bases, com M ∈ AD e N ∈ BC . Prove que P é o ponto médio de M N e que M N é igual à média harmônica
2ab
de a e b, i.e., prove que M N = .
a +b

Solução:

Exercício 4.10

1◦ parte:
A partir do trapézio ABC D traçamos a altura H e a altura entre o lado C D e M N , h, é possível perceber
x H −h x h
que os triângulos ADC ∼ AM P e D AB ∼ D M P pelo caso A A A, . Assim temos: = (1) e = (2)
b H a H
x x H −h h ab
respectivamente. Somando (1) e (2) obtemos + = + ⇒x= .
b a H H a +b
y H −h y h
Deforma análoga, BC D ∼ B N P e C B A ∼ C N P pelo caso A A A, . Assim temos: = (3) e =
b H a H
y y H −h h ab
(4) respectivamente. Somando (3) e (4) obtemos + = + ⇒y= .
b a H H a +b
ab ab
Como: x = ey= , x = y, ou seja, o ponto P é ponto médio de M N .
a +b a +b

2◦ parte:
2ab
Queremos provar que M N =
a +b
Resolução:

77
Observando os triângulos semelhantes, temos: ∆D M P ≡ ∆D AB logo,

D M DP x x DP
= = ⇒ = (1).
DA DB a a DB

Assim como ∆C P N ≡ ∆C AB logo,


CP CN y y CP
= = ⇒ = (2).
C A CB a a CA
E também ∆DPC ≡ ∆B P A logo,

DP b PC b b
= = ⇒ PC = P A (3) e DP = P B (4).
BP a PA a a

x + y DP PC
Somando (1) e (2): = + (5)
a B D AC
x b PB
Substituindo (4) em (1) obtemos: = . (6).
a a BD
y b PA
Substituindo (3) em (2) obtemos: = . (7).
a a CA
x + y b PB b PA x + y PA PB
Somando (6) e (7) obtemos = . + . ⇒ = + (8)
a a BD a C A b C A BD
Agora Somando (5) e (8):
M N M N DP PC P A P B
+ = + + +
a b B D AC AC B D
a + b PC + P A P B + P D
MN. = +
ab AC BD
a + b AC B D
MN. = + =2
ab AC B D
2ab
MN =
a +b

Exercício 4.11, página 139[IMB] Sobre o lado BC de um triângulo ABC marcamos um ponto Z. Em
←→ ←→
seguida, traçamos por B e C respectivamente as retas r e s, ambas paralelas a AZ . Se AC ∩ r = {X } e
←→
AB ∩ s = {Y }, prove que B1X + C1Y = AZ
1
.

Solução: Observe a figura abaixo:

78
Exercício 4.11

AZ ∥ r ∥ s
AC ∩ r = {X }
AB ∩ s = {Y }.
Temos que BXC ∼ ZAC pelo caso de semelhança AA, pois o AZC= YCB e ZAC = CYB pelo paralelismo. Daí,

AC
BY = YAZC = CBCZ (1)

e ainda CXB ∼ ZAB pelo caso de semelhança AA, pois XBC = AZB = BXC pelo paralelismo daí,

XC
AB = BAZX = BC
ZB (2)

.
AC
De (1) e (2) temos respectivamente: BY = YAZC = CBCZ → C Z = AZ .BC
CY e B Z = BCB.AZ 1 1
X → BX + CY =
1
AZ

Exercício 4.12, página 139[JGB] Em um trapézio ABCD, de bases AB e CD e lados não paralelos AD e BC,
seja M o ponto médio da base CD. O segmento AM intersecta a diagonal BD em F.Traçamos por F areta r,
paralela às bases. Se r intersecta os segmentos AD, AC e BC respectivamente em E, G e H, prove que E F =
FG = GH.

Primeiramente faremos um esboço da figura.

Exercício 4.12

79
Queremos provar que E F = F G = G H .
Temos as seguintes semelhanças de triângulos:

AE AF EF AF AG FG
4AEF ∼ 4ADM ⇒ AD
= AM
= DM
e ainda 4AFG ∼ 4AMC ⇒ AM
= AC
= MC
. Como DM = MC, temos:
EF = FG.

BF BG FG BM BC MC
4BFG ∼ 4BDM ⇒ BD = BM = DM e ainda 4BMC ∼ 4BGH ⇒ BG = BH = GH . Como DM = MC, temos:
FG = GH.

Portanto, E F = FG = G H .

Exercício 4.13, página 139[EVO] Seja ABC um triângulo tal que AB = c, AC = b e BC = a. Se ∠ ABC =
2∠ AC B, mostre que b 2 = c(a + c).

Solução:

Exercício 4.13

Na figura acima, D é o pé da bissetriz do lado AC. O triângulo BCD é isósceles de base BC e os triângulos
ADB e ABC são semelhantes, pois

• ∠B AD = ∠B AC

• ∠ ADB = ∠ ABC

• ∠ AB D = ∠ AC B.

Então,
AD BD AB AD +C D AB b c
= = ⇒ = ⇒ =
AB BC AC AB + BC AC a +c b

⇒ b 2 = c(a + c)

80
Exercício 4.14, página 139[JGB] Um triângulo ABC é tal que ACbB = 2B AC
b e AC = 2BC . Mostre que tal
triângulo é retângulo.

Solução: Observe o triângulo ABC abaixo.

Queremos mostrar que ABC é retângulo.

Pelo resultado do problema anterior, temos:

Daí, adaptando ao nosso problema:


p
c2 = a(2a + a) ⇒ c2 = 3a2 ⇒ c = a 3.

Note que

p
(2a)2 = a2 + c = (a 3)2 ⇒ 4a2 = a2 + 3a2 .

Pela recíproca do teorema de Pitágoras, o 4ABC é retângulo em B.

Exercício 4.15, página 139[JCR] Sejam Γ(O; R) o círculo circunscrito a um triângulo ABC e H a o pé da
altura de ABC relativa ao lado BC . Se A 0 é o simétrico de A em relação a O, prove que A A 0C ∼ AB H a .
Conclua, a partir daí, que se AB = c, AC = b e AH a = h a , então

bc
ha = .
2R

Solução: Observe a figura

81
Exercício 4.15

A 0C = A BC
Quero provar que A c b , pois são ângulos inscritos no mesmo arco capaz correspondente ao
AC .
arco d
Como AC A 0 é o ângulo que corresponde ao semicírculo de diâmetro A A 0 , temos que A H
ca B = ACb A 0 = 90◦.
Logo, pelo caso de semelhança de triângulos AA, segue-se que A A 0C ∼ AB H a
Sendo AB = c, AC = b e AH a = h a , temos:

A A0 AC A 0C 2R b bc
= = ⇒ = ⇒ Ha =
AB AH a B H a c Ha 2R

Exercício 4.18, página 139 [LOC] Seja ABC um triângulo retângulo de catetos b e c e altura h relativa à
hipotenusa. Prove que

1
h2
= b12 = c12

Solução Note que H AC ∼ ABC pelo critério AAA, portanto

b
a
=m
b
⇒ b 2 = ma (1)

82
Note também que H B A ∼ ABC

c
a
= nc ⇒ c 2 = na (2)

Finalmente, temos que H AC ∼ H B A

h
m = nh ⇒ h 2 = mn (3)

Somando os inversos de (1) e (2), temos:

n m
1
b2
+ c12 = 1
am
1
+ an = amn + amn = m+n
amn

Note que m + n = a, logo:

1 a
b2
+ c12 = amn
1
= mn

Que é exatamente o inverso de h 2 , ou seja,

1
b2
+ c12 = h12 .

Como queriamos mostrar.

83
Exercício 4.19, página 139[JEO] Dados reais positivos a e b, considere, no plano, um segmento AB de
comprimento a + b e um ponto H sobre o mesmo, tal que AH = a e B H = b. Trace um semicírculo de
diâmetro AB e, em seguida, marque o ponto C , obtido com a interseção de tal semicírculo com a reta
perpendicular a AB e passando pelo ponto H .

Exercício 4.19
p
Mostre que C H = ab

Solução: O triângulo ABC formado pela construção está inscrito em uma circunferência e o lado AB
está sobre o diâmetro, logo AĈ B = 90◦ , valendo as relações métricas de um triângulo retângulo. Assim o
quadrado da altura relativa ao vértice C é igual ao produto de a e b, ou seja:

2 p
CH = a ×b ⇒CH = ab

Exercício 4.20,página 140[TCF] Dados, no plano, segmentos de comprimento a e b, construa com régua
p
e compasso um segmento de comprimento ab.

Solução: Seja ABC um triângulo retângulo em A com catetos AB = x, AC = y e hipotenusa BC = a. Sendo


H o pé da altura relativa a hipotenusa, C H = w, B H = b e AH = h, temos pela proposição 4.9 que h 2 = ab ⇒
p
h = ab.

84
Exercício 4.20

Exercício 4.21, página 140[JGB]


Sejam M, N e P pontos respectivamnte sobre os lados BC, CA e AB de um triângulo equilátero ABC de
lado a, tais que B M = C N = AP = a3 . Mostre que o triângulo MNP também é equilátero e que seus lados são
perpendiculares aos lados de ABC.

Solução: Sendo ABC equilátero de lado a, onde B M = C N = AP = a3 .

Exercício 4.21

Queremos provar que o 4MNP também é equilátero e seus lados são perpendiculares ao lado de ABC.
p
Usando o teorema de Pitágoras no 4PMB, onde x = 3/3(a), então:

p a 2
( 3/3(a))2 = ( 2a 2
3 ) + ( 3 ) , logo P M ⊥ BC . Analogamente, P N ⊥ AB e M N ⊥ AC .

Portanto, o triângulo MNP também é equilátero e que seus lados são perpendiculares aos lados de ABC.

Exercício 4.23, página 140[JGB] Seja ABCD um trapézio de bases AB e CD e lados não paralelos AD
e BC, retângulo em A. Sabendo que AB e CD medem, respectivamente, 12 cm e 4 cm, e que ABCD é
circunscritível, calcule as distâncias dos vértices B e C ao centro do círculo inscrito em ABCD.

85
b = Bb + Cb = 180◦ . Como AB//CD, temos: OP⊥AB e OR⊥CD.
Solução: Temos que Ab + D
Segue que OP = OS = SA = AP = SD = DR = OR = r, logo S é o ponto médio de AD e RC = CQ = 4 - r e QB = PB
= 12 - r.
Vamos calcular BO e CO.
BP e BQ são tangentes à circunferência inscrita, logo BC é bissetriz de Bb. Segue que CR e CQ também são
tangentes, logo, CO é bissetriz de Cb.
B C
OQB e OQC são triângulos retângulos semelhantes, pois Bb + Cb = 180◦ ⇒ 2 = 2 = 90◦ .
OBQ = B ⇒ QOB = C e OCbR = C ⇒ COQ = B .
2 2 2 2

r 4−r 48
12−r
= r
⇒ r2 = 48 - 12r - 4r +r2 ⇒ r = 16
= 3.

Sendo r = 3, temos que:

p
(OB)2 = 32 + 92 = 9 + 81 = 90 ⇒ OB = 3 10

p
(OC)2 = 32 + 12 = 10 ⇒ OC = 10.

Exercício 4.24, página 140[NLD] A hipotenusa BC de um triâgulo retângulo ABC é dividida em quatro
segmentos congruentes pelos pontos D,E e F. Se BC = 20cm, calcule, em centímetros, o valor da soma
AD 2 + AE 2 + AF 2 .

Solução: Observe a figura baixo:

Queremos mostrar o valor da soma AD 2 + AE 2 + AF 2 . Aplicando o Teorema de Pitágoras nos triângulos


retângulos, temos:
• AD2 = (3Y )2 + X 2
•AE2 = (2Y )2 + (2X )2
•AF2 = Y 2 + (3X )2
202 = (4X )2 + (4Y )2 ⇒ 400 = 16(X 2 + Y 2 ) ⇒ X 2 + Y 2 = 25.

86
AD 2 + AE 2 + AF 2 = 14(X 2 + Y 2 ) = 350.
Logo , AD 2 + AE 2 + AF 2 = 350.

Exercício 4.25, página 140[RGA] Identifique construa com régua e compasso o LG dos pontos médios
das cordas de comprimento L de um círculo Gama (O; R) dado.
p à p ! µ ¶2
4R 2 − l 2 4R 2 − l 2 2 l
Solução: O LG é o círculo de centro O e raio , ou seja, o círculo Σ 0, 2
x =R -
2 2 2
Descrição dos passos:
l
1o ) Trace um círculo de raio R> ;
2
2o ) Marque um ponto A sobre o círculo e com abertura l centrado em A marque B sobre o círculo;
3o ) Trace o segmento AB em seguida trace a mediatriz de AB obtendo M.
4o ) Como OM AB , para cada p corda de comprimento l temos pelo teorema de Pitágoras que a distância de
4R 2 − l 2
l , ao centro O será sempre
2

Exercício 4.25

Exercício 4.26, página 140[PSM] Sejam Γ um círculo de centro O e raio R no plano . Prove que o LG dos
pontos no plano a partir dos quais podemos traçar tangentes de comprimento l a Γ é o círculo de centro O
p
e raio R 2 + l 2 .

Solução:

87
Quando traçamos a tangente l ou AC ela forma com o centro da circunferência O e o raio R. O OC a
2 p
hipotenusa do triângulo retângulo. Temos pelo Teorema de Pitágoras que OC = L 2 + R 2 → OC = l 2 + r 2
isto vale para qualquer posição em que estava o ponto A. Como queríamos mostrar.

Exercício 4.27, página 140[RCP] Um pedrestre situaldo a 25m de um edifício, o visualiza sob um certo
ângulo. Em seguida, ele se afasta mais 50m do edifício e nota que, ao assim fazer, o novo ângulo de
visualização é exatamente a metade do anterior. Calcule a altura do edifício.

Solução; Seja A o topo e B a base do edifício, assim como C e D as posições inicial e final do pedrestre.
ˆ = α.
Sejam ACˆ B = α e ADB 2
ˆ = ACˆ B − ADB
ˆ = α
Pelo teorema do ângulo externo, temos D AC 2
. Sendo o triângulo AC D isósceles de
base AD. Portanto, AC = C D = 50 e, aplicando o teorema de Pitágoras ao triângulo ABC , obtemos AB =
p p
502 − 252 = 25 3.

Exercício 4.28, página 140:[SLS] As retas r,s e t são paralelas, estando s entre r e t, de tal modo que a
distância entre r e s é 3m e a distância entre s e t é 1m. O triângulo ABC é equilátero e tem um vértice sobre
cada uma das retas r,s,t. Calcule o comprimento de seu lado.

Solução: Suponha, sem perda de generalidade, que A ∈ r , B ∈ s e C ∈ t , e sejam D e E, respectivamente, os


p
pés das perpendiculares baixadas de A e B à reta t. Agora, sendo AB = BC = l , temos que C E = l 2 − 1,C D =
p p p p
l 2 − 16 e DE = l 2 − 9. Por outro lado, não é difícil verificar que D ∈ C E de sorte que l 2 − 1 = l 2 − 16 +
p
l 2 − 9. Resolvendo tal equação, encontramos o valor de l.

Exercício 4.29, página 141[TCF] Duas torres com 30m de altura e a outra com 40m de altura, estão
situados a 50m uma da outra. Entre ambas as torres ha uma fonte, para a qual dois passarinhos partem, em
um mesmo instante e com velocidades iguais, do alto de cada torre. Sabendo que os passarinhos chegam
a fonte simultaneamente, calcule a distância da fonte a base da torre mais baixa.

Solução:

Exercício 4.29

Sabendo que os passarinho partem com velocidades iguais do alto de cada torre e chegam a fonte no
mesmo instante, concluímos que as distâncias AC e BC são iguais.

88
Aplicando o Teorema de Pitagoras temos:
x 2 = 302 + a 2 e x 2 = b 2 + 40
402 + b 2 = 302 + a 2
a 2 − b 2 = 700
E ainda temos que a + b = 50 Entao temos:
a 2 − b 2 = 700
{
a + b = 50
a − (50 − a)2 = 700
2

a 2 − 2500 + 100a − a 2 = 700


100a = 3200
a = 32
A distância da fonte a base da torre mais baixa é de 32 metros.

p
Exercício 4.30, página 141:[VT] Se os comprimentos de dois dos lados de um triângulo são 7 e 5 2, e se
o ângulo compreendido entre tais lados mede 135◦ , calcule o comprimento do terceiro lado.

Solução: Observe dos dados do enunciadoque conhecemos a medida de dois lados do triângulo e o
ângulo oposto ao lado desconhecido. logo, podemos utilizara lei dos cossenos para calcular a medida
deste lado desconhecido. A lei dos cossenos tem a seguinte equação: A 2 = B 2 + C 2 − 2AB cos x onde A é o
p
lado oposto ao ângulo x. Temos então B = 7m,C = 5 2m e x = 135◦ , logo a medida do terceiro lado do
p p
triângulo é:A 2 = 72 + (5 2)2 − 2.7.5 2. cos 135◦ ⇒ A 2 = 49 + 50 + 70 ⇒ A 2 = 169 ⇒ A = 13 metros.

Exerício 4.31, página 141[JEO] Dado um ponto P no interior de um retângulo ABCD, de diagonais AC e
2 2 2 2
BD, prove que AP +C P = B P + DP .

Solução: Observe o retângulo ABC D abaixo:


2 2 2 2
Queremos provar que AP +C P = B P + DP
Traçamos as paralelas aos lados do retângulo, passando por P, obtemos os pontos E, F, G, H.

Aplicando o Teorema de Pitágoras aos triângulos :APG, B PG, DP F e C P F , temos:

AP 2 = PG 2 + AG 2 (1)
C P 2 = P F 2 +C F 2 (2)
B P 2 = PG 2 + BG 2 (3)
DP 2 = P F 2 + F D 2 (4)

Somando (1) e (2)


AP 2 +C P 2 = PG 2 + BG 2 + P F 2 + F D 2

Mas, AG = F D = x e BG = C F = y, pelo paralelismo.


Assim

89
AP 2 +C P 2 = PG 2 + x 2 + P F 2 + y 2

B P 2 + DP 2 = PG 2 + y 2 + P F 2 + x 2

logo, AP 2 +C P 2 = BG 2 + DP 2

Suponha P 1 < P 2 , o outro caso é análogo

AP 1 AP 2
Se =
P1B P2B
,
então pelo princípio fundamental das proporções, temos AP 1 · P 2 B = P 1 B cd ot AP 2

Exercício 4.35, página 141[AA] Seja s a reta numerada e α um dos semiplanos, dentre os que s determina.
1
Para cada n ² Z , trace o círculo de raio , contido em α e tangente a s em n. Em seguida, sendo Γ1 e Γ2 dois
2
círculos já traçados e tangentes exteriomente, trace todos os círculos tangentes a s e a Γ1 e Γ2 . Repetindo
essa operação recursivamente, prove que o conjunto dos pontos de tangência de tais círculos com s está
contido no conjunto Q dos números racionais.

1
Solução: Temos os círculos de raio r = tangente a reta s nos pontos n − 1, n²Z .
2
Mostraremos que

Primeiramente, vamos encontrar o valor de x utilizando a expressão de x sendo o raio o terceiro círculo
1 1 1 1
e r e R os raios dos dois círculos temos que p = p + p , como r =
x r R 2
,
1 1 1 1
p = r + r =⇒ x =
x 1 1 8
2 2

90
Analisando o triângulo retângulo abaixo, temos:
(r + x)2 = (r − x)2 + (nk)2
r 2 + 2r x + x 2 = r 2 − 2r x + x 2 + (nk)2
=⇒ 4r x = (nk)2
p
=⇒ nk = 2rr x
1 1 1
=⇒ nk = 2 . = 2.
2 8 4
1
=⇒ nk =
2
1
Portanto, k²Q, pois a distância de n a k é , como n²Z e a soma de um inteiro com um racional é um
2
número racional.

91
Soluções do capítulo 4,página 150

Exercício 4.1, página 150[CAR] Construa com régua e compasso o triângulo ABC , conhecendo as posições
dos vértices B e C e do pé da bissetriz interna relativa a A, bem como o comprimento b do lado AC .



Solução: Seja P ∈ BC o pé da bissetriz interna relativa a A. Constrói-se o ponto Q ∈ B C BC , tal que
BC
= BP .
QC PC
←→
Para a construção de Q, basta fazer duas retas r e s paralelas, diferentes de BC , tais que B ∈ r e C ∈ s.
Fixe um segmento arbitrário, de comprimento u, e marque sobre r um ponto X tal que B X = 2u e sobre
S o pontos Y e Z tais que C Y = C Z = 3u. Marque Y e Z de tal forma que X e Y estejam em um mesmo
←→ ←→ ←→
semiplano, daqueles determinados pela reta BC . Faça as retas X Y e X Z e marque o ponto Q com uma das
intersecções dessas retas com a reta over l e f t r i g ht BC .
O vértice A pode ser obtido como a interseção do círculo de diâmetro AC = b e centro C com o círculo
de diâmetro PQ.

Exercício 4.1

Exercício 4.5, página[EVO] ABCD é um quadrilátero de diagonais AC e BD, inscrito em um círculo Γ.


Mostre que existe P ∈ Γ tal que P A + PC = P B + P D.

Solução: Trace o círculo de Apolônio relativo ao arco AC . O ponto P é a interseção de tal círculo com o
arco AD que não contém B.

Exercício 4.7, página 150[JCR] São dados um natural n e um círculo Γ de raio 1. Se AB é um diâmetro de
Γ, prove que existem pontos C 1 , ....C n ∈ Γ tais que AC i , BC i ∈ Q, 1 ≤ i ≤ n.

92
Solução: AB é diâmetro de Γ. Como o raio de Γ = 1, AB = 2. Seja C i um ponto sobre Γ e t um racional
entre 0 e 1 (t ∈ Q, 1 ≤ i ≤ n.)
Pela propriedade da densidade dos racionais entre dois reais quaisquer, temos que exitem infinitos "t "
racionais entre dois reais quaisquer.
2 − 2t 2
Então qualquer que seja o n dado, teremos como escolher n t 0 s 0 < t < 1, tal que AC i = e BC i =
1+ t2
4t
.
1+ t2

t ∈ Q ⇒ AC i , BC i ∈ Q
Alem disso,
4 − 8t 2 + 4t 4 + 16t 2
(AC i )2 + (BC i )2 = = 4 = (AB )2
1 + 2t 2 + t 4
⇒ ABC i é retângulo em C i , o que completa a prova. Já que C i é um ângulo inscrito num semicírculo.

Exercício 4.11, página 151[JEO] Em um triângulo não equilátero ABC , de ortocentro H e circuncentro
O, prove que a mediatriz do segmento HO e a bissetriz interna relativa ao lado BC intersectam-se sobre o
círculo circunscrito ao triângulo AHO.

Solução:

Exercício 4.11

Seja P o ponto em que a bissetriz interna relativa ao lado BC intersecta o círculo circunscrito ao triângulo
AHO. O teorema do ângulo inscrito garante que H ÂP = H ÔP e O ÂP = O Ĥ P . Por outro lado, H ÂP = O ÂP
e H ÔP = O Ĥ P . Portanto, H P = OP e assim, P pertence à mediatriz do segmento HO.

93
Soluções do cápitulo 4, página 161

Exercício 4.7, página 162[TCF] Complete a prova do Teorema 28. Mais precisamente, mostre que, se A A 0 ,
B B 0 e CC 0 são cevianas paralelas de um triangulo ABC , com isogonais A A 00 , B B 00 e CC 00 , respectivamente,
entao A A 00 , B B 00 eCC 00 sao concorrentes ou paralelas.

Solução: E suficiente mostrar que, se A A 00 e CC 00 concorrem num ponto P ∈ B B 00 . Para tanto suponha,
sem perda de generalidade, que A 0 ∈ BC e B 0 ∈ C A \ AC , de sorte que C 0 ∈ B A \ AB (os demais casos sao
analogos). Se B Ab A 00 = α, a isogonalidade de A A 0 e A A 00 garante que C Ab A 0 = α, ao passo que o paralelismo
de A A 0 e CC 0 , juntamente com a isogonalidade de CC 0 e CC 00 , fornece C 00CbB = ACCb 0 = C Ab A 0 = α. Portanto,
temos B CbP = B CC b 00 = α = B Ab A 00 = B AP
b ,de sorte que o quadrilatero B AC P e inscritivel; segue, entao, que
P BC
b = P AC b . Por fim, tal igualdade, em conjuncao com a isogonalidade de A A 0 e A A 00 e o paralelismo de
A A 0 e B B 0 , nos da, P BC b = A BbB 0 . Segue que B B 0 e B P sao isogonais em relacao a B , de
b = A 0 AB
b = A 00 AC
maneira que P ∈ B B 00 .

Exercício 4.11, página 162[AJS] Prove o teorema de Pappus8 : são dados dois ternos de pontos colineares
←→ ←→ ←→ ←→ ←→ ←→
A, B , C e A 0 , B 0 , C 0 . Se AB 0 ∩ A 0 B = {F }, AC 0 ∩ A 0C = {E } e BC 0 ∩ B 0C = {D}, então os pontos D, E e F são
colineares.

Solução: Demonstração: Seja o hexágono não convexo AB 0C A 0 BC 0 com os vértices A, B e C sobre uma
reta e A 0 , B 0 e C 0 sobre outra reta. Seja E o ponto de intersecção dos lados opostos C A 0 e AC 0 , D o ponto
de intersecção dos lados opostos BC 0 e C B 0 e F ponto de intersecção dos lados opostos A 0 B e AB 0 . Seja
F o ponto de intersecção das retas BC 0 e AB 0 , H ponto de intersecção das retas C A 0 e AB 0 , e I , o ponto
←→ ←→ ←→
de intersecção das retas C A 0 e BC 0 . Estamos assumindo que as retas C A 0 , BC 0 e AB 0 formam um triângulo
GHI.
Aplicando o Teorema de Menelaus aos cinco trio de pontos: {E ,C 0 , A}, {C , D, B 0 }, {A 0 , B, F }, {C , B, A}
0
HE H A0
e {A 0 ,C 0 , B 0 } em relação ao triângulo G H I , temos: EI · CIC0G · GA
AH = 1, HC
CI
ID
· DG · GA
AH = 1, A0 I
I B GF
· BG · F H = 1,
HC IB GA H A0 0 0
CI
· BG · AH
=1e A0 I
· CIC0G · BGB
0 H = 1. Dividindo o produto das três primeiras equações pelo produto das
HE I D GF
duas últimas, obtemos, após cancelamento, EI
· DG · F H = 1, o que nos dá novamente, pelo Teorema de
Menelaus, aplicado ao trângulo G H I , que os pontos E , D e F são colineares.

Exercício 4.11

94
Exercício 4.13, página 162[AA] Em um triângulo escaleno ABC , traçamos a altura AH a e, em seguida,
←→ ← → ←→ ←→
baixamos as perpendiculares H a D e H a E respectivamente às retas AB e AC (D ² AB e E ² AC ); em seguida,
←→ ← →
marcamos o ponto P de interseção das retas DE e BC . A partir das alturas relativas aos vértices B e C de
←→
ABC , construímos, de maneira análoga, os pontos Q α AC e R ² AB . Mostre que os pontos P , Q e R são
colineares.

Exercício 4.13

Solução: Considere que os pontos P,Q e R sejam colineares, pela reciproca do Teorema de Menelaus,
temos:
M P NC QR
. . =1
MC N R PQ
8 5 QR
=⇒ . . =1
4 20 PQ
Aplicando a Lei dos Cossenos, temos:
3
42 = 62 + 52 − 2.6.5.cosx =⇒ cosx =
4
2 3 p p
QR = 12 + 20 − 2.12.20. =⇒ QR = 184 = 2 46 (1)
2 2
4
2 2 2 9
5 = 6 + 4 − 2.8.6.cos y =⇒ cos y =
16
2 9 p
PQ = 82 + 62 − 2.8.6. =⇒ PQ = 46 (2)
16
8 5 QR
Substituindo (1) e (2) em =⇒ . . = 1 temos:
4 20 PQ
p
8 5 2 46
=⇒ . . p =1
4 20 46

Logo, os pontos P, Q e R são colineares.

95
Soluções dos exercícios do cápitulo 4, página 176

Exercício 4.1, página 176[CAR] Prove a proposição 30.

←→
Solução: Proposição 30: Se A, B , C e P são pontos distintos no plano com B ∈ AP e C ∈/ AB , então P A.P B =
2 ←→
PC se, e só se, o círculo que passa pelos pontos A, B e C for tangente à reta PC em C .

Exercício 4.1

Analisando os triângulos APC e B PC , temos que ∠ APC = ∠B PC , PC é lado comum e que ∠PC A =
∠P BC , pois determinam o mesmo arco na circunferência, portanto, são iguais. Sendo assim, os triângulos
APC e B PC são semelhantes pelo caso ALA, logo:
PA
PC = PC
P B ⇒ P A.P B = PC
2

Exercício 4.5,página 176[EVO] Seja ABC um triângulo isósceles de base BC = a e h a o comprimento da


a 2 +4h 2
altura relativa à base. Sendo R o raio do círculo circunscrito a ABC, mostre que R = 8h .

Solução:

96
Exercício 4.5

Aplicando o Teorema das Cordas às cordas AD e BC, temos

M A × M D = M B × MC

a2
⇒ (2R − h)h =
2

a 2 + 4h 2
⇒R = .
8h

Exercício 4.7, página 176[JCR] São dados uma reta r e pontos, A e B , num mesmo semiplano dos determinados
por r . Construa com régua e compasso, todos os círculos passando por A e B e tangentes à reta r .

Solução: 1- Trace a mediatriz de AB .


2- Marque o ponto O sobre a mediatriz de AB .
3- Trace o círculo Γ com centro O e raio O A.
←→ ←→
4- Trace a reta PO 5-Obtenha os pontos A 0 e B 0 , simétricos de A e B respectivamente em relação a reta OP
6-Construa o quadrilátero AB B 0 A 0 . Observe que tal quadrilátero é inscritível no círculo Γ.
7- Trace as diagonais AB 0 e B A 0 do quadrilátero AB B 0 A 0 .
8- Marque o ponto C na interseção das diagonais.
←→
9- Trace uma reta s perpendicular a OP passando por C .

97
10- Marque os pontos D e E , interseção da reta s com o círculo Γ.
←→ ← →
11- As retas P D e PE são as retas tangentes do círculo Γ traçadas por P .
O LG dos pontos de contato das tangentes traçadas a Γ a partir de P são os pontos D e E , obtidos no passo
10.

Exercício 4.7

Exercício 4.11, página 176[JEO] Em um triângulo acutângulo ABC , a reta suporte da altura relativa a
AC intersecta o círculo de diâmetro AC em M e N . A reta suporte da altura a AB intersecta o círculo de
diâmetro AB em P e Q. Prove que os pontos M , N , P e Q são concíclicos.

Exercício 4.11

98
Solução Ao traçar as retas suportes da altura, elas irão se encontrar em H que é o or t ocent r o. Observando
os círculos Γ e ∆ temos M H .H N = AH .H H a (1) e Q H .H P = AH .H H a (2) respectivamente. Ao comparar
(1) e (2) obtemos M H .H N = Q H .H P que pelo teorema das cordas garante que é possível construir um
quadrilátero inscrito em uma circunferência contendo os pontos M , N , P e Q, logo, são concíclicos.

99
Solução dos exercícios do capítulo 5

Exercício 5.1[ACR] ABCD é um retângulo de lados AB = 32m e BC = 20m. Os pontos E e F são, respectivamente,
os pontos médios dos lados AB e AD. Calcule a área do quadrilátero AECF.

Solução: Dada a figura abaixo:

Exercício 5.1

A área do retângulo ABCD é dada por A ABC D = 20.32 = 640M 2 . Calculando agora a área do 4E BC e
do 4C DF temos 160m 2 para cada triângulo. Para calcularmos a área do quadrilátero AECF, teremos que:
A ABC D − 4ABC − 4C DF = 640 − 160 − 160 = 320M 2 .

Exercício 5.2, página 184[CA] No paralelogramo ABC D, de diagonais AC e B D, marcamos o ponto E ,


←→ ←→
sobre o lado AD, tal que B E ⊥ AD. Se B E = 5 cm, BC = 12 cm e AE = 4 cm, calcule a área do triângulo
EC D.

Solução:
←→ ←→ ←→
1. Seja F o pé da perpendicular baixada de C à reta AD. Dado que B E ⊥ AD, com E o pé da perpendicular,
segue que os triângulos AB E e C DF são congruentes pelo caso CH, de modo que B E = C F = 5 cm e
AE = DF = 4; além das hipotenusas AB = C D (hipótese) e reto em E e F , respectivamente.

←→ ←→ ← → ← →
2. Note que o triângulo BC E é retângulo em B . Dos fatos que BC ∥ AD e B E ∥ C F , tem-se que a projeção
←→
de BC sobre a reta AD é igual a E F . Segue que o quadrilátero formado BC F E é retângulo.

100
3. Observa-se que a área do retângulo BC F E é também a soma das aréas dos triângulos BC E , EC D e C DF .
Logo,

A(BC F E ) = A(BC E ) + A(EC D) + A(C DF )


A(EC D) = A(BC F E ) − ³A(BC E´) − ³A(C DF´)
A(EC D) = (BC · B E ) − BC 2· B E − DF 2·C F
A(EC D) = (12 · 5) − 122· 5 − 42· 5 = 20
¡ ¢ ¡ ¢

4. Portanto, a área do triângulo EC D é 20 cm2 .

Exercício 5.3[IMB] Seja ABCD um quadrado de lado 1, E o ponto médio de BC e F o de CD. Sendo G o
ponto de interseção de DE e AF, calcule a área do triângulo DFG.

Solução: Observe o quadrado ABCD abaixo.

Exercício 5.3

Queremos calcular a área do triângulo DFG.


Peo caso de congruência LAL, temos que 4ADF ∼
= 4DGF ;Daí , 4ADF ∼ 4DGF . Aplicando o Teorema de
Pitágoras:
p
5
(AF )2 = (AD)2 + (DF )2 → AF = 2 .
Pela semelhança de triâgulos:

DF
AF
= DG
AD
GF
= DF
p p
A(DGF
5
5
=k → A(ADF
= K 2 → (DGF
(ADF
=( 5
5 2
) = 15 . Como (ADF) = 41 , t emos : (DE F ) = 14

101
Exercício 5.4, página 185 [LOC] Se ABC é um triângulo equilátero com lados de comprimento a, prove
que:

p
a 3
(a) as alturas de ABC medem 2
p
a2 3
(b) A(ABC ) = 4

Solução (a) Como AB = BC = AC = a.


Seja M o ponto médio de AB e MC a altura relativa ao lado AB .
a
Então AM = 2
e MC = h.
Note que o triângulo AMC é retângulo em M , portanto:

a 2 = ( a2 )2 + h 2
2
h 2 = a 2 − a4 = a 2 (1 − 14 )
2
p
a 3
h 2 = 3a4 ⇒ h = 2 .

Como o triângulo é equilátero segue-se que todas as alturas serão iguais.

(b) Como foi provado na proposição 5.4 a área do triângulo é dada por:

ah a
A(ABC ) = 2

Portanto
p
a a2 3
A(ABC ) = 2
p
a2 3
A(ABC ) = 4

Exercício 5.5 , página 185:[MHV] Seja ABCD um quadrado de lado 1cm e E um ponto interior de ABCD,
tal que o triãngulo ABE seja equilátero. Calcule a área do triângulo BCE.

Solução: Dado o quadrado de lado 1cm, sendo E o vértice do triângulo equilátero ABE, temos que a área
1 12
do triângulo BCE pode ser calculada como A(BC E ) = 2
→ A(BC E ) = 14 . Outro modo de determinar a área
de BCE é : A(BC E ) = A(ABC D) − A(AB E ) − A(C DE ) − A(ADE )

102
Exercício 5.6[NLD] ABCD é um quadrado de lado 1cm e AEF um triângulo equilátero, com E ∈ BC e
F ∈ C D. Calcule a área de AEF.

Solução:

Exercício 5.6

Para encontrarmos a área do triângulo equilátero AEF, observe que BE = DF = X e CE = CF = 1 - X. Seja AE=
AF= EF = Y ( o lado de triângulo AEF). Pelo Teorema de Pitágoras, em ECF, temos : y 2 = 2(1 − x 2 ) (I).
Por outro lado, também pelo teorema de Pitágoras, em ABE , temos: y 2 = x 2 + 1 (II). De (I) e (II), temos :
x 2 + 1 = 2(1 − x)2 ⇒ x 2 + 1 = 2 − 4x + 2x 2 ⇒ x2 − 4x + 1 = 0. Resolvendo
p
a equação do 2°grau, obtemos
p p 2 y2 3 p
x = 2 + 3 e y = (2 + 3) + 1. Agora, pelo fato de que a área de AEF = 4 ⇒ a área de AEF = (2 3 + 3).

Exercíco 5.8, página 185[RCP] Generalizando o item (b) do problema anterior, mostrando que, se P é
um ponto no interior de um polígono regular A 1 A 2 ...A n , então a soma das distâncias de P às retas suportes
dos lados de A 1 A 2 ...A n independe da posição de P.

Solução: Com relação a A 1 A 2 ...A n , seja O o centro, l o comprimento do lado e a a distância comum de O
aos lados.
←−−−→
Seja, ainda, C i o pé da perpendicular baixada de P à reta A i A i +1 (A n+1 = A 1 ). A partir das igualdades temos:

103
Pn Pn
i =1 A(A i A i +1 ) = i =1 A(P A i A i +1 )

l ·PC i
n · l2a =
Pn
i =1 2
−→ PC 1 + PC 2 + ... + PC n = na.

Exercício 5.9, página 185:[SLS] O triângulo ABC tem lados a,b,c. As alturas correspondentes a tais lados
são respectivamente iguais a h a , h b , h c . Se a + h a = b + h b = c + h c , prove que ABC é equilátero.

Solução: Denotando por S a área de ABC, temos h a = 2S 2S 2S


a , h b = b , h c = c . Portanto, h a , h b , h c . Se a + h a =
b + h b = c + h c se, e só se, a + 2S
a
= b + 2S
b
+ c + 2S
c
. Agora, a + 2S
a
= b + 2S
b
equivale a (a − b)(2S − ab) = 0 ou,
ainda a =b ou ab= 2S; analogamente, a + 2S
a
= c + 2S
c
se, e só se, a= c ou ac= 2S. Se c 6= a, b , então ac = 2S
ac ah a
= bc, de sorte que a=b. Então 2 =S= 2 , de forma que c = h a e daí, ABC é retângulo em B, portanto,
b = AC > BC = ao que é uma contradição.

Exercício 5.10, página 185[VT] Seja ABC um triângulo retângulo de área 1m 2 . Calcule a a´rea do triângulo
←→ ←→
A’B’C’, onde A’ é o simétrico de A em relaçao a BC , B’ é o simétrci de B em relação a AC e C’é o simétrico de
←→
C em relação a AB .

Solução: Podemos supor, sem perda de generosidade, que ABC é retângulo em A. Como BB’ e CC’ se
←→ ← → ←−→ ← →
intersectam em seus respectivos pontos médios, BC’B’C’ é um paralelogramo. Como A A 0 ⊥ BC e B 0C 0 ∥ BC ,
←→ ←−→ ←→
temos A A 0 ⊥ B 0C 0 . Marque os pontos H e H’, respectivamente de interseçao de A A 0 com BC e B’C’. Uma
vez que ABC ∼ = A 0 BC ∼= AB 0C 0 , temos A 0 H 0 = A 0 H + AH + AH 0 = 3AH . Portanto ,A(A’B’C’) = 1 B 0C 0 .A 0 H 0 =
2
3
2
BC .AH = 3A(ABC ) = 3m 2 .

104
Soluçãos dos exercício do capítulo 5 , página 197

Exercício 5.2, página 197[MHV] : Sejam ABCD um paralelogramo e EFG um triângulo cujos vértices estão
situados sobre os lados de ABCD. Prove que

A(ABC ) Ê 2A(E F G).

Solução: Consideremos, inicialmente , o caso em que os vértices E,F e G do triângulo coincidam com três
dos vérices do quadrilátero, digamos A,B e C, respectivamente. Como o quadrilátero ABCD (ou EFGD) é um
paralelogramo , ele está dividido em dois triângulos congruentes EFG e EDG, palo caso LLL de congruência
de triângulos. Sendo assim, A(ABCD)= 2A(EFG).

Mantendo fixa a base EF e deslocando o ponto G sobre o lado CD, teremos um novo triângulo EFG, mas
com a mesma área do anterior, uma vez que possuem mesma base e mesma altura, onde A(ABCD) =
2A(EFG).
Consideramos, agora , o triângulo EFG da figura abaixo, em que E ∈ AB , F ∈ C D e G ∈ AD.

Temos que a base FG do triângulo EFG é menor que a base AC do triângulo ABC, que é a diagonal maior do
paralelogramo ABCD, também temos que a altura EFG relativa a FG é menor do que a altura do triângulo

105
ABC relativa a AB, que a altura do paralelogramo. Neste, e nos casos similares, teremos sempre A(ABC ) Ê
2A(E F G). Logo, concluímos que A(ABC ) Ê 2A(E F G).

Exercício 5.3, página 197[MHV] : Três formigas, inicilamente situadas em três dos vértices de um retângulo,
se movem uma por vez e de acordo com a seguinte regra: quando uma formiga se move, ela se desloca na
direção paralela à formada pelas outras duas formigas .É possível que e algum instante as formigas estejam
situadas nos pontos médios de retângulo original?

Solução: Seja ABCD um retângulo no qual estão situadas as formigas F 1 , F 2 e F 3 respectivamente nos
vértices A,B e D. Os vértices que inicialmente estão as três formigas (A,B e D) determinam um triângulo
AB x AD
retângulo de área 2
.

Como as formigas se movem uma por vez e cada uma delas só pode se mover na direção paralela a direçãoformada
plas outras duas. Temos que as possíveis posições das três formigas sempre formarão triângulos de área
igual ao triângulo ABD.
Para justificar tal fato fixamos as formigas localizadas nos pontos B e D e deslocamos a formiga localizada
no ponto A até o ponto A’ pertencente a reta BC.

106
Os lados BC e AD do retângulo ABCD são paralelos segue então qua ambos possuem a mesma medida.
Ainda sabemos que é fato que ∠ A 0 AB e ∠B DC são ângulos correspondentes e, portanto congruentes. Daí
temos a congruência dos triângulos A’AB e BDC por ALA já que os ângulos ∠ A 0 B A e ∠BC D são ângulos
retos.
AB x AD
Logo segue que A 0 B = BC implica qua A(A’AB) = A(BDC) = 2
.
Agora notemos que qualquer que seja o ponto da reta AA’ que a formiga estiver teremos um triângulo de
base BD e com altura igual, pois, cmo as retas AA’e BD são paralelas e então a distância entre elas é igual
em qualquer ponto.
Caso as F 1 , F 2 e F 3 estivessem cada uma em um ponto médio do retângulo elas estariam determinando
AB
2 x AD
2 AB x AD
um triângulo que possui área igual a 2
= 8
.Concluímos então que não é possível que as formigas
estejam em algum momento situadas nos ponto médios de ABCD.

Exercício 5.5, página 197:[MHV] Em um hexágono convexo ABCDEF, temos AB ∥ C F,C D ∥ B E e E F ∥ AD.
Prove que as áreas dos triângulos ACE e BDF são iguais.

Solução: Queremos provar que A(ACE) = A(BDF)

i) Como AB ∥ C F , temos A(ABC)=A(ABF)


ii) Como C D ∥ B E , temos A(CDE) = A(BCD)
iii) Como E F ∥ AD, temos A(AEF)= A(DEF).
Por outro lado,
1) A(ABCDEF) = A(ACE) +A(ABC)+A(CDE) +A(EFA).
2) A(ABCDEF)= A(BDF) + A(BCD) + A(DEF) + A(FAB)
Igualando 1) e 2) temos: ACE + ABC + CDE + EFA = BDF + BCD+ DEF + FAB.
De i), ii) e iii), concluímos : A(ACE) = A(BDF).

107
Exercício 5.6, página 197[MHV] O trapézio ABCD, de bases AB e CD e lados não paralelos AD e BC , é
retângulo em A. Se BC = C D = 13cm e AB = 18cm, calcule a altura e a área do trapézio, assim como a
←→
distância do vértice A à reta BC .

Solução:

Marcando o ponto E ∈ AB ,tal que C E ⊥AB e C E ∥ AD. Temos que DC ∥ AB , assim AE = 13cm e EB =
5cm. No 4C E B , pelo teorema de Pitágoras, h 2 = 132 − 52 → h = 12cm.Agora veja F o ponto de BC, tal que
AF ⊥BC . Queremos calcular AF . Aplicando o teorema de Pitágoras , em ADC, ACF e AFB, respectivamente
, temos:
•x 2 = 132 + 122 → x 2 = 313.
•x 2 = y 2 + z 2 → y 2 = 313 − z 2 .
•182 = y 2 + (13 − 2)2 → z = 158 2 158 2 216
26 → y = 313 − ( 26 ) → y = 13 ≈ 16, 6cm.

Exercício 5.7, página 208:[ACR] Sejam A,B e C pontos colineares, com B ∈ AC , e Γ, Γ1 e Γ2 semicírculos de
diâmetros respetivamente iguais a AC,AB,BC, situados em um mesmo semiplano dos determinados pela
←→
reta AC . A reta r, perpendicular a AC passando por B, intersecta Γ em D. Sendo S a área da porção de Γ que
2
BD
é exterior a Γ1 e Γ2 , calcule a razão S
.

Solução: Dadas as semicircunferências Γ, Γ1 e Γ2 de raios R, R 1 E R 2 e diâmetros AC,AB e BC respectivamente,


e sabendo que B ∈ AC e r ⊥ AC , bem como D ∈ r r Γ. Temos:AC = AB + BC → 2R = 2R 1 + 2R 2 → R =
2 2
π.(R 1 )
− π.(R2 2 ) ⇒ S = π2 .(2R 1 R 2 ) ⇒ S = π.R 1 R 2 .
2
R 1 + R 2 .(2) A área S é dada por: S = A2r − A2r 1 − A2r 2 = π.R
2 − 2
Considerando o ângulo A DC b , temos que este é reto, pois é inscrito a circunferência Γ de diâmetro AC, logo
2 2
b = 90◦ . De onde decorre,pelas relações métricas do triângulo retângulo: B D = AB .BC ⇒ B D =
, A DC
2 2 2 2
2R 1 .R 2 ⇒ B D = 4R 1 .R 2 , de (2) temos: B D = 4.S. π1 ⇒ B SD = π4 . Portanto BD
S = π4 .

108
109
Solução dos exercícios do capítulo 6

π 3π
Exercício 6.1, página 221[ACR] Marque sobre o ciclo trigonométrico as extremidades dos arcos de 4
, 4
,
3π −2π −3π −4π −5π
2 , −π, 3 , 2 , 3 e 2 radianos.

Solução:

Exercício 6.2, página 221[CA] Marque, em cada quadrante do ciclo trigonométrico, os sinais do seno,
cosseno e tangente.

Solução
Consideremos no plano cartesiano, o ciclo trigonométrico representado pelo círculo Γ, centrado na origem
_
O(0, 0), com raio 1 e comprimento 2π. Para c ∈ R, em sentido trigonométrico, os arcos AP (de início no
ponto A e extremidade final em P ) de c radianos, teremos que os sinais do seno, cosseno e tangente são
como dados a seguir.
Sabendo que sin c = ordenada de P (P 2 ) e cos c = abscissa de P (P 1 ),
_
• c está no 1◦ quadrante ⇐⇒ P ∈ AB ⇐⇒ 0 + 2kπ ≤ c ≤ π2 + 2kπ, com k ∈ Z
- sin c: para c no primeiro quadrante, o ponto P está acima do eixo das abscissas e sua ordenada é positiva
⇒ sin c > 0;
- cos c: para c no primeiro quadrante, o ponto P está acima do eixo das abscissas e sua projeção horizontal
está sobre O A ⇒ cos c > 0;
sin c
- tan c = cos c
, com c 6= 0 ⇒ tan c > 0.

110
_
π
• c está no 2◦ quadrante ⇐⇒ P ∈ B A 0 ⇐⇒ 2 + 2kπ ≤ c ≤ π + 2kπ, com k ∈ Z
- sin c: para c no segundo quadrante, o ponto P está acima do eixo das abscissas e sua ordenada é positiva
⇒ sin c > 0;
- cos c: para c no segundo quadrante, o ponto P está acima do eixo das abscissas, porém, sua projeção
horizontal está sobre O A 0 ⇒ cos c < 0;
sin c
- tan c = cos c
, com c 6= 0 ⇒ tan c < 0.

_
• c está no 3◦ quadrante ⇐⇒ P ∈ A 0 B 0 ⇐⇒ π + 2kπ ≤ c ≤ 3π
2
+ 2kπ, com k ∈ Z
- sin c: para c no terceiro quadrante, o ponto P está abaixo do eixo das abscissas, logo, a ordenada é negativa
⇒ sin c < 0;
- cos c: para c no terceiro quadrante, o ponto P está abaixo do eixo das abscissas e sua projeção horizontal
está sobre O A 0 ⇒ cos c < 0;
sin c
- tan c = cos c
, com c 6= 0 ⇒ tan c > 0.

111
_

• c está no 4◦ quadrante ⇐⇒ P ∈ B 0 A ⇐⇒ 2 + 2kπ ≤ c ≤ 2π + 2kπ, com k ∈ Z
- sin c: para c no quarto quadrante, o ponto P está abaixo do eixo das abscissas, logo, a ordenada é negativa
⇒ sin c < 0;
- cos c: para c no quarto quadrante, o ponto P está abaixo do eixo das abscissas, porém, sua projeção
horizontal está sobre O A ⇒ cos c > 0;
sin c
- tan c = cos c
, com c 6= 0 ⇒ tan c < 0.

−π
Exercício 6.3,página 221[IMB] Calcule o seno, o cosseno e a tangente (se existir) dos arcos de π , 2 , 4π,
7π −3π
2
e 2
radianos.

Solução:

Exercício 6.3

2π 3π 5π 7π 4π 5π 5π 3π
Exercício 6.4, página 221 [LOC] Calcule o seno, o cosseno e a tangente de 3 , 4 , 6 , 6 , 3 , 4 , 3 , 2 ,

4 .

Solução Utilizando o corolário 6.13: Para todo c ∈ R, temos:

sen(c) = sen(π − c)
−cos(c) = cos(π − c)

e o fato que

α 0 π/6 π/4 π/3 π/2


p p
seno 0 1/2 2/2 3/2 1
p p
cosseno 1 3/2 2/2 1/2 0
p p
t ang ent e 0 3/3 1 3 ∞

112
conseguimos obter todos os valores para, seno, cosseno e tangente desejados no problema.

α 2π/3 3π/4 5π/6 7π/6 5π/4 4π/3 3π/2 5π/3 7π/4


p p p p p p
seno 3/2 2/2 1/2 −1/2 − 2/2 − 3/2 −1 − 3/2 − 2/2
p p p p p
cosseno −1/2 − 2/2 − 3/2 − 3/2 − 2/2 −1/2 0 1/2 2/2
p p p p p
t ang ent e − 3 −1 −1/ 3 1/ 3 1 3 ∞ − 3 −1

113
Solução dos exercícios do capítulo 6, página 226

Exercício 6.1, página 226[NLD] Prove as fórmulas da proposição 15 para os desenvolvimentos de sin(a −
b) e tan(a − b).

Solução: Utilizando que: sin θ = − sin θ e cos (−θ) = cos θ .


Seja: sin(a − b) = cos ( π2 − a + b) ⇒ cos ( π2 − a) cos b + sin ( π2 − a) sin (−b) ⇒ sin a cos b − cos a sin b. 2.

Para: tan (a − b) ⇒ tan (a − b) = sin (a−b)


cos (a−b
sin a cos b−cos a sin b (tan a−tan b)
= cos =
a cos b+sin a sin b 1+tan a tan b
2.

Exercícios 6.2, página 226[PSM] Calcule o seno, o cosseno e a tangente de 15◦ .

p p p p p p p
2 3 2 1 6 2 6+ 2
Solução: cos 15◦ = cos (45 − 30) = cos 45◦ . cos 30◦ + sin 45◦ . sin 30◦ = .
2 p 2
+ . =
2 2 p 4p 4
+ = 4
.
p p p p
2 3
sin 15◦ = sin (45 − 30) = sin 45◦ . cos 30◦ − cos 45◦ . sin 30◦ = .
2 2
− . = 46 − 42 = 6−4 2 .
2 1
2 2
p
1− 3 p
tan 45 −tan 30 ◦ ◦ (3− 3)2
tan 15◦ = tan (45 − 30) = 1+tan 45◦ . tan 30◦ =
3p
= 6 .
1+1. 33

Exercício 6.3, página 226[RCP] Para a ∈ R, prove que


(
cos 2a =
1-2sen2 a

Solução: Sabendo que sen2 a + cos2 a = 1 e que cos 2a = cos2 a − sen2 a, temos:
1. Tomando cos2 a = 1 − sen2 a,
cos 2a = (1 − sen2 a) − sen2 a = 1 − 2 sen2 a.
Logo, cos 2a = 1 − 2 sen2 a.
2. Considerando sen2 a = 1 − cos2 a,
cos 2a = cos2 a − (1 − cos2 a = 2 cos2 a − 1
Portanto, cos 2a = 2 cos2 a − 1

Exercício 6.4, página 226:[SLS] Use as fórmulas do corolário 16 e do problema anterior para calcular o
seno, o cosseno e a tangente de um ângulo de 22◦ 300 .

Solução: 22◦ 300 = 45 2


. Basta agora usar a fómula do arco metade para a tangente :
q
tan ( x2 ) = 1−cos x
1+cos x
.
p
2
cos 45 = 2s
p
p
r
2
1− 2−p2
tan ( 45
2
)= p2 = .
1+ 22 2+ 2
p p p q p p
r p
(2− 2)(2−
p 2) )(2 − 2) = 6−4 2
tan ( 45
2
)= 2
= 3 − 2 2.
(2+ 2

114
Exercício 6.5, página 226:[VT] Em cada um dos casos a seguir, resolva as equações dadas para x ∈ R:
a) sin x = 0.

Solução: A equação sin x = 0 tem no intervalo [0◦ , 360◦ ]([0, 2π]) duas soluções:o seno anula-se em 0◦ e
180◦ (0 e πr ad ). No universo das amplitudes,temos sin x = 0 ⇔ x = 0◦ +k.180◦ , k ∈ Z ⇔ x = k.180◦ , k ∈ Z(x =
kπ, k ∈ Z.
b) cos x = 0

SoluçãO; A equação cos x = 0 tem no intervalo [0◦ , 360◦ ]([0, 2π]) duas soluções: o cosseno anula-se em 90◦
e 270◦ ( π2 e 3π
2 ) rad. No universo das amplitudes, temos: cos x = 0 ⇔ x = 90◦ +k.180◦ , k ∈ Z(x = π2 +kπ, k ∈ Z).
c) tan x = 0

Solução: A equação tan x = 0 tem no intervalo [0◦ , 360◦ [([0, 2π[) duas soluções : 0◦ e 180◦ (0 e π rad) - os
zeros do seno em [0◦ , 360◦ [. No universo das amplitides, temos : tan x = 0 ⇔ x = 0◦ + k.180◦ , k ∈ Z ⇔ x =
k.180◦ , k ∈ Z(x = kπ, k ∈ Z).

Exercício 6.7, página 226[PSM] Em cada um dos casos a seguir,resolva as equações dadas para x ∈ R,
p
−1 3
quando A = 2 ,A = 2 e A = −1:
a) sin x = A
b) cos x = A

Solução: sin x = A; sin x = −1


2
→ sin( 3π
2
± kπ
3
) = −1
2
com k ∈ Z.
p p
3 π kπ 3
sin x = A → sin 2
→ sin( 2
± 6
) = 2
.
sin A = −1 → sin 3π
2 = −1.
b) cos x = 2 → cos (π ± kπ
−1 −1
3 ) =p 2 .
p
cos x = 23 → cos (2π ± kπ
6
) = 23 ; k ∈ Z.
cos x = −1 → cos (π ± 2kπ) = 1; k ∈ Z.

p
Exercício 6.8, página 226[PCM] Resolva a equação tan x = A quando A = 3 e A = 1.

p p p
Solução: t anx ± 3; (60◦ e 120◦ ) → tan x = 3 → t an( 3π
4 ± 5π
12 ) = 3

tan x = 1(45◦ e 225◦ ) → tan( 3π
4 ± 2 )=1

115
Solução dos exercícios do capítulo 7

Exercício 7.1[ACR] Prove que , no espaço , existem quatro pontos não coplanares. Quantos planos distintos
ficam determinados por três desses quatro pontos?

Solução: Suponha quatro planos α , β , γ, θ. Sendo que , β, γ , θ são paralelos e concorrentes a α. Assim
teremos, uma reta na intersecção de cada plano:
• A reta r ∈ β ∩ α.
• A reta s ∈ γ ∩ α.
• A reta t ∈ θ ∩ α.
Dados os quatro pontos: A,B,C,D pontos, onde nenhum deles pertencem a r ou a s oua t. Mas, o ponto A ∈
β, o ponto B ∈ γ , o ponto C ∈ θ e o ponto D ∈ α. Assim, teremos que os pontos A,B,C,D não são coplanares.
Temos que, três pontos determinam um plano. Assim, usando combunação de 4 pontos tomados 3 a 3 ,
um totalizando quatro planos. Observando a figura abaixo, podemos concluir que esses pontos no espaço
correspondem, aos vértices de um tetraedro cujas faces são os quatro planos, os planos são: ABC, ACD,
ABD, CDB.

Exercício 7.1

Exercício 7.2, página 257[CA] Ache o maior número possível de regiões em que três planos dividem o
espaço.

Solução
Considerando três planos α, β e γ. Se forem coincidentes, teremos o mesmo resultado de α inicialmente
que é apresentando a seguir. Sejam então os três planos distintos.
Sem perda de generalidade, consideremos o plano α. Temos, por definição, α determina no espaço dois
conjuntos convexos disjuntos. Com β 6= α, em decorrência da definição 7.1., existem duas possibilidades:
α ∩ β = ∅ ou α ∩ β 6= ∅.
α ∩ β = ∅ indica α k β (1). Para tal, se β é paralelo à α, independente de sua posição no espaço, ele separa
em 2 um dos semiplanos formados inicialmente por α. Determinando assim 3 regiões.
α∩β 6= ∅ indica β concorrente à α. Neste caso, β dividirá em 2 cada um dos dois semiplanos formados por
α, totalizando assim 4 regiões. Dado o que objetivamos, usaremos este resultado.

116
α e β paralelos (esquerda) e concorrentes (direita).

Para um terceiro plano γ, pela proposição 7.6., temos quatro situações a verificar junto ao último resultado
favorável obtido da relação entre α e β concorrentes.
a) γ paralelo a um dos planos: como α e β são concorrentes, γ poderá ser paralelo somente a um deles.
Em construção semelhante ao apresentado em (1), o plano γ secciona algum par de regiões já existentes,
formando assim mais 2 semiplanos, num total de 6 regiões.
b) γ não é paralelo a nenhum dos outros dois planos: neste caso decorrem três possibilidades para a relação
entre os planos.
i ) α, β e γ contêm uma mesma reta (α ∩ β ∩ γ 6= ∅ ). Assim, γ secciona apenas duas regiões que antes eram
não adjacentes. Disto, formam-se 2 semiplanos em cada uma das duas anteriores, totalizando 6 regiões
(ver Figura 7.9, página 256).
i i ) α, β e γ são dois a dois concorrentes e as retas de interseção são duas a duas paralelas (2). Das quatro
regiões formadas por α e β, todas são adjacentes duas a duas. O plano γ, ao seccionar de forma que não há
ponto comum aos três planos, estará concorrendo em três regiões duas a duas adjacentes. Logo, formam-
se 3 semiespaços dois a dois adjacentes, totalizando 7 regiões (ver Figura 7.10, página 257).
i i i ) α, β e γ são dois a dois concorrentes e as retas de interseção concorrem em um único ponto. Em
processo semelhante a (2), ao seccionar α e β em apenas um ponto comum aos três, γ também estará
concorrendo nas regiões duas a duas adjacentes. Porém, desta forma, estará seccionando simultaneamente
as quatro regiões duas a duas adjacentes, formando-se então 4 semiespaços, donde totaliza-se 8 regiões
(ver Figura 7.10, página 257). Logo, observa-se deste ser o maior número possível de regiões em que três
planos dividem o espaço.

Exercício 7.3[IMB] Dados, no espaço, um ponto A e uma reta r , com A 6∈ r , prove que existe uma única
reta s, paralela a r e passando por A.

Solução: Observe a imagem abaixo:

117
Exercício 7.3

Sejam A e B os pontos de interseção de t com r e s respectivamente. Se as retas r e s não fossem paralelas


então teriam um ponto comum e os ângulos α e β também seriam diferentes.
Para as retas r e s serem paralelas α e β (ângulos correspondentes) logo α = β. Logo r ∥ s ⇐⇒ r e s possuir
a mesma inclinação.

Exercício 7.4, página 257 [LOC] Sejam r , s e t retas no espaço, tais que r ∥ s e s ∥ t . Prove que r ∥ t .

Solução Se r , s e t estiverem contidas em um mesmo plano, nada há a fazer. Senão, sejam α = (r, s),
β = (s, t ) e γ = (A, r ), onde A é um ponto sobre a reta t . Se β ∩ γ = t 0 , então, pela proposição 6, temos r , s e
t 0 paralelas. Mas, como A ∈ t , t 0 ⊂ β e t , t 0 ∥ s, temos t = t 0 ; em particular, r ∥ t .

Exercício 7.5, página 257:[MHV] Sejam α e α0 dois planos distintos e ABC e A’B’C’ triângulos situados,
←→ ←−→ ← → ←−→ ← → ←−→
respectivamente, em α e α0 . Se as retas AB e A 0 B 0 , BC e B 0C 0 , AC e A 0C 0 forma duas a duas concorrentes,
prove que seus pontos de interseção são colineares.

Soluçã:; Os pontos A,B,C,e D,E,F estão situados em planos α e β distintos,respectivamente.


←→ ←→
Como M pertence às retas AB e DE , que estão contidas em α e β, respectivamente ( Teorema 2.1), o ponto
M pertence aos planos α e β.
Analogamente, os pontos N e P pertencem a α e β. Como α e β são distintos e possuem os pontos M,N
e P em comum,esses planos não são paralelos nem coincidentes. Logo, os planos α e β são secantes e se
intersectam segundo uma reta que contém M,N e P. Portanto, os pontos M,N e P são colineares.

Exercício 7.6[NLD] Prove a seguinte versão do teorema de Thales no espaço: sejam α e α0 planos paralelos,
A um ponto que não pertence a α ou α0 e X e Y pontos de α. Se as retas AX e AY intersectam α0 respectivamente
AX AY
nos pontos X’ e Y’, então = .
AX 0 AY 0

Solução: A reta r 1 corta os planos nos pontos A 1 , B 1 e C 1 e r 2 corta os mesmos planos nos pontos A 1 , B 2 e
C 2 . Pelo ponto A 1 de r 1 traçamos uma reta r 20 paralela a r 2 , que corta os três planos nos pontos A 1 , B 20 e C 20 .
As retas r 1 E r 20 determinam um plano que corta β e γ segundo as retas paralelas B 1 B 20 E C 1C 20 . Logo, pelo
A1 B1 B 1C 1 A 1C 1
Teorema de Talespara retas paralelas, temos A 1 B 20
= B 20 C 20
= A 1C 20
. Mas A 1 B 20 = A 2 B 2 , B 20 C 20 = B 2C 2 , e A 1C 20 =

118
A 2C 2 , por serem segmentos retas paralelas compreendidos entre retas paralelas. Logo temos:
A1 B1 B 1C 1 A 1C 1
A2 B2 = B 2C 2 = A 2C 2 .

Teorema de Tales para planos paralelos

Exercício 7.7, página 258[PSM] Sejam α e α0 planos paralelos, A um ponto que não pertence a α ou α0
←→ ← → ←→
e XYZ um triângulo em α. Se as retas AX , AY e AZ intersectam α0 respectivamente em X 0 , Y 0 e Z 0 , prove
que X 0 Y 0 Z 0 ∼ X Y Z .

Solução: Observe a figura abaixo:

Exercício 7.7

Temos dois palnos α e α0 paralelos entre si , o 4X Y Z ∈ α. Traçando retas transversais dos planos α e α0
saindo de X,Y,Z vértices do 4X Y Z passando por um ponto A 6∈ α e A ∈ α será gerado um outro 4X 0 Y 0 Z 0 ∈

119
ao plano α. Como pela propriedade de retas paralelas cortadas por transversais eram ângulos congruentes
entre si , logo os 4X Y Z e 4X 0 Y 0 Z 0 são semelantes pelo caso AAA.
Obs: Congruência L.A são iguais semelhantes AA = AA com lados porporcionais.

120
Soluções dos exercício do cápitulo 7 ,página 268.

Exercício 7.1, página 268[RCP] Ache o maior inteiro positivo n par o qual exitam no espaço retas r 1 , r 2 , ..., r n ,
duas a duas ortogonais ou perpendiculares.

Solução:

Dadas no espaço as retas r 1 , r 2 , ..., r n , duas a duas perpendiculares ou ortoganias. Fixando-se no espaço
o ponto A e traçando paraelas a estas retas de modo que todos passem por A, temos:
1. Dado um plano qualquer, que contenha A, exitem neste plano apenas duas retas concorrentes em A e
que formam ângulo de 90ř, ou seja que são perpendiculares.
2. Dadas duas retas concorrentes em A é única a reta que passa por este ponto e é perpendicular as restas
concorrentes e ao plano que as contém.
3. Portanto, de acordo com 1 e 2, o número máximo de retas no espaço, que são duas a duas perpendiculares
ou ortogonais é n = 3.

Exercício 7.2, página 268:[SLS] Sejam α e β planos dados e r uma reta também dada, tal que r ⊥ α. Prove
que r ⊥ β se, e só se, α ∥ β.

Solução: Suponha, inicialmente, que r ⊥ β. Trace planos distintos γ e δ contendo r, e sejam s e s’ (resp. t
e t’) as interseções de γ (resp. de δ) com α e β, respectivamente. Então r ⊥ α ⇒ r ⊥ s, t e r ⊥ β ⇒ r ⊥ s 0 , t 0 .
Mas , como s, s 0 ⊂ γ e t , t 0 ⊂ δ segue que s ∥ s 0 e t ∥ t 0 . Portanto, segue da proposição 7.4 que α e β são planos
paralelos. Reciprocamente , se α ∥ β, então podemos escolher retas concorrentes s,t ⊂ α e s 0 , t 0 ⊂ β, tais que
s ∥ s 0 e t ∥ t 0 . Mas, como r ⊥ α, temos r ⊥ s, t ,e, daí, r ⊥ s 0 , t 0 . Segue da proposição 7.8 que r ⊥ β.

Exercício 7.3, página 268:[VT] Sejam dados um ponto A e um plano α, com A ∈ α. Prove que existe uma
única reta que passa por A e é perpendicular a α.

121
Solução: Tome um plano β paralelo a α e construa, com a reta r que passa por A e é perpendicular a β.

122
Solução dos exercícios do capítulo 8

Exercício 8.1[ACR] Seja ABCD um tetraedro dado. Traçamos por A uma reta r, que intersecta a face BCD
e forma ângulos iguais com as arestas AB, AC e AD. Sendo P o ponto onde r intersecta a esfera circunscrita
a ABCD , prove que

AB = AC = AD ⇒ P B = PC = P D.

Solução: Sejam ABCD um tetraedro regular e P o pé da altura baixada de A a face BCD. Então os triângulos
ABP, ACP e ADP são retângulos em P tais que AB=AO=AD. Portanto, pelo caso especial dos triângulos
retângulos segue que BP=CP=DP.
1) O triângulo PBO é perpendicular em O.
2) O triângulo PCO é perpendicular em O.
3) O triângulo PDO é perpendicular em O, são congruentes, então conclui-se que BP=CP=DP.

Exercício 8.3:[IMB] Prove que uma pirâmide de vértice V e base A 1 A 2 ...A n é inscritível se, e só se, o
polígono A 1 A 2 ...A n for inscritível. Nesse caso, mostre que a esfera circunscrita à mesma é única.

Solução: Sendo uma pirâmide inscrita em uma esfera de centro O temos que essa pirâmide para ser
inscrita precisa ser uma pirâmide regular. Temos OV um dos raios da esfera Σ ligando o ponto O (centro da
esfera) a cada vértice de base A 1 , A 2 , A 3 , ....., A n de uma pirâmide regular, também temos raio da esfera Σ.
Sendo a esfera Σ única garantindo que qualquer pirâmide regular com seus vértices tangenciado a esfera e
cada vértice da pirâmide ligado ao centro da esfera gera o raio da esfera circunscrita a essa pirâmide.

Exercício 8.4, página 294 [LOC] Seja C um cone de revolução de vértice V , raio R e altura h. Trace, à
distância d < h do plano de base do cone, um plano que o intersecta segundo um círculo Γ0 , de raio R 0 . A
porção do cone situada entre sua base e o plano α é denominada o tronco de cone de bases Γ e Γ0 (ou raios
R e R 0 ) e altura d . Prove que a área lateral A de um tal tronco de cone é dada por
q
0
A = π(R + R )d
0
1 + ( R−R
d )
2

Solução A área do tronco de cone em questão é igual a π(R g − R 0 gp0 ), onde g e g 0 são as geratrizes dos
g g0 d 2 +(R−R 0 )2
cones dado e daquele de base Γ0 e vértice V . Mostre que R
= R0
= R−R 0
; em seguida substitua as
expressões para g e g 0 obtidas a partir daí para obter a igualdade desejada.

Exercício 8.5, página 294[MHV] Sejam ABCD um tetraedro regular e M e N pontos distintos, situados
respectivamente sobre as faces ABD e ACD. Mostre que M N , B N e C M são os comprimentos dos lados de
um triângulo.

123
Solução: Dado o tetraedro regular ABCD e os pontos M e N, temos : O plano mediador de BC é o plano
α = (ADP ) onde P é o ponto médio de BC. Portanto , α deixa M e N em semiespaços opostos, de forma que
que B M < C M . Segue que : M N < B M + B N < C M + B N . No entanto se M’ é o simétrico de M em relação
ao plano (ACD), temos M N = M 0 N E C M = C M 0 . Portanto, C M = C M 0 < B M 0 < B N + M 0 N = B N + M N .
Analogamente , temos : B N = C M + M N .Logo, M N , B N e C M são os comprimentos dos lados de um
triângulo.

Exercício 8.8. página 295[RCP] 8.8 Seja ABC D um tetraedro qualquer e E , F,G, H , I e J pontos respectivamente
sobre AB , BC , C A, AD, B D, e C D. Sabendo que
AE · B E = B F · C F = AG · CG = AH · D H = B I · D I = C J · D J
mostre que os pontos E , F,G, H , I e J estão situados sobre a mesma esfera.

P
Solução: Dado um tetraedro ABC D, seja (0; R) a esfera circunscrita a este. Seja k o valor comum dos
produtos. Temos:
A secção de pelo plano (AOB ) é um equador Γ de , logo pelo teorema das cordas aplicado a Γ, obtemos:
P P
2 p
k = AE · B E = R 2 − OE −→ OE = R 2 − k.
De modo analogo, podemos obter:
p
OF = OG = OH = OI = O J = OE = R 2 − k.
p
Portanto, os pontos E , F,G, H , I e J estão todos situaos sobre a esfera de centro O e raio r = R 2 − k.

Exercício 8.9, página 295:[SLS] Sejam ABCD um tetraedro qualquer e M,N,P e Q os baricentros das faces
BCD, ACD, ABD e ABC, respectivamente. Prove que :
a) os segmentos AM, BN, CP E DQ se intersectam em um único ponto G, denominado o baricentro do
tetraedro.
AG BG
b) = = CG = DG = 3.
GM GN GP GQ

Solução: Se R é o ponto médio de CD, temos M ∈ B R e N ∈ AR; daí, AM e BN são cevianas do triângulo
ABR, de sorte que se intersectam em um ponto G. Como RN = 13 = RM , temos RMN ∼ RBA por LAL e, assim
RA RB
←−→ ← → ←−→ ← →
, M N ∥ AB E M N = 13 AB . Por sua vez , o paralelismo de M N e AB garante que G M N ∼ G AB e, então ,
MG MN MG 0
AG = = 31 . Analogamente, comcluímos que AM e CP se intersectam em um ponto G’,tal que AG 0 = 13 ;
AB
portanto , segue do problema 1.2,página 147 G = G 0 . Da mesma forma, mostramos que DQ também passa
por G.

Exercício 8.10, página 295:[VT] Em uma pirâmide quadrangular regular,a altura mede h e os lados da
base medem a . Se α é o plano que passa por dois vértices adjacentes da base e pelo ponto médio da altura,
calcule, em função de a e h, a área da seção da pirâmide determinada por α.

124
Solução: Sejam V o vértice da pirâmide , ABCD sua base, O o pé de sua altura, M o ponto médio de VO
e α = (AD M ). Entã , O é o ponto de interseção de AC e BD. Ademais, como A, M ∈ (AV C ), a semirreta
−−→ −−→
AM intersecta VC em um ponto P; analogamente, D M intersecta VB em um ponto Q. Portanto, AQ, DP
e PQ estão contidos em α , de sorte que a seção é o quadrilátero ADPQ. Afirmamos que tal quadrilátero
a
é um trapézio isósceles de bases a e 3
.Para tanto, seja l o comprimento das arestas laterais da pirâmide.
Aplicando o teorema de Menelaus ao triângulo COV, com os pontos colineares A, M e P, obtemos V P = 21 C P
e, daí, V P = 13 V C = 31 ; analogamente , V P = 13 . Portanto : (i) BQ = C P , de modo que ABQ ∼
= DC P por LAL e,
1 a ←→ ← → ←
→ ←→
assim, AQ = DP . (ii) V PQ v V C B por LAL, de maneira que PQ = 3 C B = 3 e PQ ∥ BC ; mas, como BC ∥ AD,
←→ ←→
cocluímos a partir daí que PQ ∥ AD. Agora , se d é o comprimento da altura de ADPQ, o teorema de
AD−PQ 2 2
Pitágoras fornece d 2 + ( 2
) = AQ . Agora aplicando, a Relação de Stewart ao triângulo ABV, obtemos
2 2
+3a 2
AQ = 4l 9 e, daí, o resultado do exemplo 8.1 fornece

2
+3a 2 2
d 2 = 4l 9
− a9 = 94 (a 2 + h 2 ).

Por fim,
p
A(ADPQ) = d2 (AD + PQ = 94 a a 2 + h 2 .

Exercício 8.12, página 295[CA] Se S é a soma das áreas das faces de um tetraedro ABC D, prove que
1 2 2 2 2 2 2
S≤ p (AB + AC + AD + BC + B D +C D ).
2 3

Solução
Seja um tetraedro ABC D. Aplicando a desigualdade de Weitzenböck (problema 6.21., página 239) a cada
uma das faces de ABC D, temos

Exercício 8.12
p 2 2 2
(I) 4 3 A(ABC ) ≤ AB + BC + AC
p 2 2 2
(II) 4 3 A(AB D) ≤ AB + AD + B D
p 2 2 2
(III) 4 3 A(AC D) ≤ AC + AD +C D
p 2 2 2
(IV) 4 3 A(BC D) ≤ BC + B D +C D .

125
Da soma de (I), (II), (III) e (IV), segue
p
4 3 (A(ABC ) + A(AB D) + A(AC D) + A(BC D)) ≤
2 2 2 2 2 2 2 2 2 2 2 2
(AB + BC + AC ) + (AB + AD + B D ) + (AC + AD +C D ) + (BC + B D +C D )
p 2 2 2 2 2 2
4 3 (S) ≤ 2AB + 2AC + 2AD + 2BC + 2B D + 2C D
p 2 2 2 2 2 2
4 3 S ≤ 2 (AB + AC + AD + BC + B D +C D )
p 2 2 2 2 2 2
2 3 S ≤ AB + AC + AD + BC + B D +C D
1 2 2 2 2 2 2
S≤ p (AB + AC + AD + BC + B D +C D )
2 3

Exercício 8.13, página 295[RCP] 8.13 Estenda a fórmula da mediana de Euler (cf. problema 6.12, página
238) para tetraedros, da seguinte forma: se ABC D é um tetraedro e M E N são, respectivamente, os pontos
médios de AB e C D, então
2 2 2 2 2 2 2
N N = 41 (AC + AD + BC + B D − AB −C D )

Solução:

Do triângulo AB N , temos:
2 2 2 2
+B N )−AB
M N = 2(AN 4
Dos triângulos BC D e AC D, segue:
2 2 2 2
B N = 2·(B D +BC )−C D
4
2 2 2 2
2·(AD +AC )−C D
AN = 4
Substituindo B N e AN em M N , obtemos:
2 2 2 2 2 2 2
+AC )−C D
2 2·[ 2·(AD 4 + 2·(B D +BC
4
)−C D
]−AB
MN = 4
Logo,

126
2 2 2 2 2 2
+2AC +2B D +2BC −2C D
2 2·[ 2AD 4 ]−AB
MN = 4
2 1 2 2 2 2 2 2
M N = · (AD + AC + B D + BC − AB −C D )
4
2 2 2 2 2 2 2
Portanto, M N = 41 · (AC + AD + BC + B D − AB −C D )

127
Solução dos exercícios do capítulo 8, página 301.

Exercício 8.1, página 301:[ACR] Prove que um prisma de bases A 1 A 2 .....A n e A 01 A 02 ...A 0N é inscritível se,e só
se, ele for reto e suas bases forem polígonos inscritíveis. Nesse caso, prove que a esfera que o circunscreve
é única.

Solução: Dado um prisma, suponha que ele é inscritível em uma esfera Γ de centro O. Como Γ circunscreve
também cisrcunscreve a pirâmide de vértice A’1, e base A 1 A 2 A 3 ...A n , sendo a base de inscrita em um
círculo de R 1 , é única e esfera Γ que contém o ponto A 01 e o círculo R 1 . Agora suas bases estão inscritas
nos círculos definidos pelas insterseções de Γ com os planos que os contém. Por outro lado, A 1 A 0A 1 A 02 é
um paralelogramo inscrito em um círculo, logo é um retângulo . Analogamente, todas as demais faces são
retângulos , portanto , o prisma é reto. Reciprocamente, se o prisma for reto e tiver bases inscritíveis em
círculos de centros de P e P’. Tomando o segmento P P 0 , sendo o ponto médio deste segmento. Existe uma
unica esfera que contém ao mesmo tempo os círculos que contém as bases e esta tem raio 12 P P 0 .

128
Solução dos exercícios do capítulo 9

Exercício 9.2, página 316[CA] Sejam dados uma esfera Σ(O; R) e um real positivo d , com d < R. Seccione
Σ por um plano α, situado à distância d de O. A calota esférica de altura R −d em Σ é a porção de Σ situada
no semiespaço oposto de α em relação a O. Calcule, em função de R e d , a área de tal calota esférica.

Solução

Exercício 9.2

Considerando 2R como a altura de Σ(O, R), temos que em função de R e d , a altura da calota é dada por
R − d . Do resultado do exemplo 9.1., podemos estabelecer a seguinte relação entre a área A(Σ) da esfera e
área A(c) da calota.

A(c)
A(Σ)
= HhcΣ
A(c)
4πR 2
= R−d
2R
2
A(c) 2R = 4πR (R − d )
A(c) = 2πR (R − d )

Portanto, em função de R e d , a área da calota esférica é dada por A(c) = 2πR (R − d ).

Exercício 9.4, página 316 [LOC] Se ABC D é um tetraedro regular inscrito numa esfera de centro O e raio
R, calcule as medidas dos ângulos do triângulo esférico de vértices A, B e C .

Solução Observe que A(ABC ) = 14 A(Σ) = πR 2 . Agora, aplique o teorema de Girard.

129
Soluções dos exercícios do cápitulo 9 página 325

Exercício 9.1, página 325:[MHV] Sejam C um conjunto convexo e A e B pontos de C, sendo A um ponto
interior. Porve que todos os pontos de segmento AB, coma possível exceção de B, também não são interiores.

Solução: Seja B(A;R) uma bola aberta contida em C. Se C ∈ AB \AB e r = BC .R mostre que B(C; r ) ⊂ C
AB

Exercício 9.2,página 325[NLD] Se A é um conjunto de pontos so espaço, prova que ∂A é um conjunto


fechado.

Solução: Seja A ∈ ∂A. Dado ² > 0, existe x ∈ ∂A tal que x ∈ (a², a + ²). E como x ∈ ∂A, existem infinitos
elementos de X em (x − ², x + ²), onde ² = min(|x − (a ± ²)|). Assim, infinitos elementos de A pertencem à
(a − ², a + ²) ⊃ (x − γ, x + γ). Isso implica que a ∈ ∂A. Concluimos que ∂A ⊃ ∂A.

Exercício 9.3, página 325[PSM] Prove que não há poliedro convexo com exatamente 7 arestas.

Solução: Sendo : V (vértices), A (arestas), F (faces). Temos então que pela relação de Euler V − A + F =
2 → V + F = 2 + A → V + F = 2 + 7 → V + F = 9 . Lembrando que o valor mínimo das faces para termos um
poliedro é 4, então temos as seguintes possibilidades.
3 f 3 +4 f 4 +5 f 5 +....+n f n
F = 4 e V=5 , neste caso quando f = 4 temos um tetraedro de faces triangulares como 2
.
Assim A = 3.4
2 → A = 6, portanto não podemos ter um poliedro convexo para F=5 e V=4, temos um tetraedro
sobrando uma face. Isso irá aocntecer para F = 6 e V = 3 , F = 7 e V = 2 , F = 8 e V = 2. Assim não é possível
ter um tetraedro com exatamente 7 arestas.

Exercício 9.4, página 325[RCP] 9.4 Prove que todo poliedro convexo tem um número par de faces com
números ímpares de lados.

Solução: Seja um poliedro convexo em que: F 3 representa o número de faces triangulares,


F 4 representa o número de faces quadrangulares,
F 5 representa o número de faces pentagonais,
F 6 representa o número de faces hexagonais,
....
....
....
Então F = F 3 + F 4 + F 5 + F 6 + ... (1)
Sendo cada aresta comum a duas faces, teremos:
2A = 3F 3 + 4F 4 + 5F 5 + 6F 6 +... (2)
Tese F 3 + F 5 + F 7 + ...é par
De fato, da relação (2) temos:
3F 3 + 4F 4 + 5F 5 + 6F 6 + 7F 7 + ... = 2A

130
−→ F 3 + F 5 + F 7 + ... = 2A − 2F 3 − 4F 4 − 4F 5 − 6F − 6 − ...
−→ F 3 + F 5 + F 7 + ... = 2(A − F 3 − 2F 4 − 2F 5 − 3F 6 − 3F 7 − ...
o que prova a tese.

Exercício 9.6, página 325:[VT] Se cada vértice de um poliedro convexo é ligado a todos os outros por
arestas, prove que o poliedro é um tetraedro.

Solução: Sejam V > 3 o número de vértices e P um dos vértices do poliedro.Como P é adjacente aos outros
V-1 vértices, os quais são, por sua vez, dois a dois,adjacentes, concluímos que o poliedro possui exatamente
à !
V −1
(V − 1) + (V − 3) = 2V − 4 faces e (V − 1) + = 12 (V 2 − V ) arestas. Então, segue da relação de Euler que
2
à !
2
4
V −7V +12 = 0 e, daí, V =4. Portanto , F=4, e A=6. Como = 6, segue que os vértices do poliedro são dois
2
a dois adjacentes;mas, como só há quatro vértices,concluímos que o poliedro é um tetraedro.

131
Solução dos exercícios do capítulo 10

Exercício 10.2, página 345[CA] Prove o corolário 5: "Seja C um cilindro sólido de revolução, de raio R e
altura h, então V (C ) = πR 2 h."

Solução
Para fins de demonstração, consideremos que a base e a altura de um cilindro sólido são a base e a altura
do cilindro que constitui sua fronteira, respectivamente. Seja C um cilindro sólido de revolução de raio R
e altura h que possui uma de suas extremidades em um plano α. Temos que C ∩ α 6= ∅ e seja o disco D a
região formada. Logo, D é base de C .
Seja P um paralelepípedo de mesma altura h que C e de base B um paralelogramo tal que sua área seja
igual à área de D. Suponhamos que os dois sólidos tenham as bases em α e estão contidos em um mesmo
semiespaço, dos que α determina.
Para um plano β paralelo a α, que secciona um dos sólidos, decorre da igualdade das alturas do cilindro
e paralelepípedo que C ∩ β 6= ∅ se, e só se, P ∩ β 6= ∅. Quando ocorre, temos que β secciona C e P e
as secções C 0 e P 0 , respectivamente, têm áreas iguais, pois são congruentes às respectivas bases. Deste
resultado, segue do Princípio de Cavalieri II então que V (C ) = V (P ).
Por fim, da proposição 3 temos que V (P ) = A(B )h, logo,

V (C ) = V (P ) = A(B )h = A(D) h = (πR 2 ) h = πR 2 h

Portanto, o volume V (C ) de um cilindro sólido C é dador por

V (C ) = πR 2 h

Exercício 10.3[IMB] Se ABCD é um tetraedro trirretângulo em A, com AD = a, B D = b e C D = c, mostre


que :
abc
V (ABC D) = 6 .
abc
A partir daí, mostre que a altura de ABCD em relação à face ABC mede 2A(ABC ) .

Solução:

Exercício 10.4, página 345 [LOC] Seja ABC D um tetraedro isósceles cujas arestas medem a, b e c. Mostre
que
p
1
ν(ABC D) = p (a 2 + b 2 − c 2 )(a 2 + c 2 − b 2 )(b 2 + c 2 − a 2 ).
6 2

Solução Use o teorema de realização 12 para calcular o volume de um tetraedro isósceles ABC D como a
diferença entre o volume do paralelepípedo reto retângulo associado a ele e a soma dos volumes de quatro
tetraedros trirretângulos congruentes.

132
Exercício 10.5, página 345:[SLS] Se ABCD é um tetraedro de área total S e r é o raio de sua esfera inscrita,
prove que :

V (ABC D) = Sr
3
.

Em seguida, utilize a fórmula acima para mostrar que:


abc
a) Se ABCD é trirretângulo em A, com AD = a, B D = b e C D = c, então r = ab+bc+ca+2A(ABC ) .
b) Se ABCD é isósceles com arestas de comprimentos a,b e c,então
p
r= p 1 (a 2 + b 2 − c 2 )(a 2 + c 2 − b 2 )(b 2 + c 2 − a 2 ).
2 2A(ABC )

Solução: Se T = ABC D e I é o centro de sua esfera inscrita, decomponha T nos tetraedros ABCI, ABDI,ACDI
e BCDI. Em seguida, aplique a fórmula da proposição 10.7 a cada um de tais tetraedros, e some os resultados
assim obtidos.Para os itens (a) e (b) utilize a primeira parte, juntamente com os resultados dos problemas
1.3 e 1.4.

Exercício: 10.6[NLD] Sejam P um ponto interior de um tetraedro isósceles e d 1 , d 2 , d 3 e d 4 as distâncias


de P às faces do mesmo. Prove que:
a) O valor de d 1 + d 2 +d 3 + d 4 independe da posição de P no interior do tetraedro.
b) Quando o tetraedro for regular temos d 1 + d 2 + d 3 + d 4 = h, onde h é a altura do tetraedro.

Solução: Dado o ponto P interior ao tetraedro regular, conforme mostra a figura abaixo, devemos mostrar
que PE + P F + PG + P H = h. Onde h é a altura do tetraedro.
De fato, a soma dos volumes das pirâmides: PABC, PBCD, PABD e PACD é igual ao volume da pirâmide
ABCD.
1
Sendo S a área de uma face do tetraedro, temso que: 3
SPE + 13 SP F + 13 SPG + 13 SP H = 13 Sh ⇒ PE + P F +
PG + P H = h . Constante.

Tetraedro

Exercício 10.9, página 345:[SLS] Um plano passa pelo vértice de um cone de revolução e forma , com o
p
plano de sua base, um ângulo de 45◦ .Sabemos que o plano intersecta a base do cone de comprimento 2 3,
a qual subentende um ângulo central de 60◦ em relação ao centro da base. Calcule o volume do cone.

133
Solução: Sejam V o vértice do cone, O o centro de sua base, AB a corda em que o plano intersecta tal
p
b = 60◦ e V MO
base e M o ponto médio de AB. Então AB = 2 3, AOB c = 45◦ . De posse de tais dados,calcule
sucessivamente o raio da base do cone, OM e V O.

Exercício 10.10, página 346:[VT] Um triângulo retângulo é rotacionado em relação a cada um de seus
lados, obtendo três sólidos de revolução.Qual desses sólidos tem maior volume?

Solução: Observe que dois desses sólidos são cones sólidos, enquanto o terceiro é a união de dois cones
sólidos com uma base comum e interiores disjuntos.

134

Você também pode gostar